Download as docx, pdf, or txt
Download as docx, pdf, or txt
You are on page 1of 46

Prayer Before the Exam

O Lord, before You lie my strength and my


University of San Agustin weakness; Preserve the one, heal the other. Before
You lie my knowledge and my ignorance. Come to
COLLEGE OF COMMERCE my assistance as I prepare for this test. Help me to
remember all that is important. Let me remember
Department of Accountancy You, let me understand You, let me love You.
AE 19 Increase these things in me until You refashion me
to Your will. Amen.
MANAGEMENT ADVISORY SERVICES (Inspired by: St. Augustine, The Trinity XV, 51)
WORKING CAPITAL FINANCE
THEORIES
1st Semester, 2020-2021

WORKING CAPITAL POLICY


. Which of the following statements is incorrect about working capital policy? (M)
a. A company may hold a relatively large amount of cash if it anticipates uncertain sales levels in the coming year.
b. Credit policy has an impact on working capital since it has the potential to influence sales levels and the speed with which
cash is collected.
c. The cash budget is useful in determining future financing needs.
d. Holding minimal levels of inventory can reduce inventory carrying costs and cannot lead to any adverse effects on profitability.
e. Managing working capital levels is important to the financial staff since it influences financing decisions and overall profitability
of the firm.
Brigham
Conservative Working Capital Policy
*. Compared to other firms in the industry, a company that maintains a conservative working capital policy will tend to have a (D)
a. Greater percentage of short-term financing.
b. Greater risk of needing to sell current assets to repay debt.
c. Higher ratio of current assets to fixed assets.
d. Higher total asset turnover.
RPCPA 0595
1
. As a company becomes more conservative with respect to working capital policy, it would tend to have a(n)
a. Increase in the ratio of current liabilities to noncurrent liabilities.
b. Decrease in the operating cycle.
c. Decrease in the quick ratio.
d. Increase in the ratio of current assets to noncurrent assets. CMA 1290 1-23
2
. As a company becomes more conservative in its working capital policy, it would tend to have a(n)
A. Decrease in its acid-test ratio.
B. Increase in the ratio of current liabilities to noncurrent liabilities.
C. Increase in the ratio of current assets to units of output.
D. Increase in funds invested in common stock and a decrease in funds invested in marketable securities. CMA 1296 1-8

. Ski Lifts Inc. is a highly seasonal business. The following summary balance sheet provides data for peak and off-peak
seasons (in thousands of dollars):
Peak Off-peak
Cash $ 50 $ 30
Marketable securities 0 20
Accounts receivable 40 20
Inventories 100 50
Net fixed assets 500 500
Total assets $690 $620

Spontaneous liabilities $ 30 $ 10
Short-term debt 50 0
Long-term debt 300 300
Common equity 310 310
Total claims $690 $620
From this data we may conclude that (M)
a. Ski Lifts has a working capital financing policy of exactly matching asset and liability maturities.
b. Ski Lifts’ working capital financing policy is relatively aggressive; that is, the company finances some of its permanent assets
with short-term discretionary debt.
c. Ski Lifts follows a relatively conservative approach to working capital financing; that is, some of its short-term needs are met
by permanent capital.
d. Without income statement data, we cannot determine the aggressiveness or conservatism of the company’s working capital
financing policy. Brigham

Aggressive Working Capital Policy


*. A firm following an aggressive working capital strategy would (M)
a. Hold substantial amount of fixed assets.
b. Minimize the amount of short-term borrowing.
c. Finance fluctuating assets with long-term financing.
d. Minimize the amount of funds held in very liquid assets. RPCPA 1091
3
. The working capital financing policy that subjects the firm to the greatest risk of being unable to meet the firm’s maturing obligations
is the policy that finances (E)
a. Fluctuating current assets with long-term debt.
b. Permanent current assets with long-term debt.
c. Permanent current assets with short-term debt.
d. Fluctuating current assets with short-term debt. CMA 1295 1-2
4
. Since Marsh, Inc. is experiencing a sharp increase in sales activity and a steady increase in production, the management of Marsh
has adopted an aggressive working capital policy. Therefore, the company's current level of net working capital
A. Would most likely be the same as in any other type of business condition as business cycles tend to balance out over time.
B. Would most likely be lower than under other business conditions in order that the company can maximize profits while
minimizing working capital investment.
C. Would most likely be higher than under other business conditions so that there will be sufficient funds to replenish assets.
D. Would most likely be higher than under other business conditions as the company's profits are increasing. CMA 0689 1-11

Aggressive vs. Conservative Working Capital Policy


5
. Clay Corporation follows an aggressive financing policy in its working capital management while Lott Corporation follows a
conservative financing policy. Which one of the following statements is correct?
A. Clay has a low ratio of short-term debt to total debt while Lott has a high ratio of short-term debt to total debt.
B. Clay has a low current ratio while Lott has a high current ratio.
C. Clay has less liquidity risk while Lott has more liquidity risk.
D. Clay's interest charges are lower than Lott's interest charges. CMA 1284 1-22
WORKING CAPITAL MANAGEMENT
Appropriate Level of Working Capital
6
. Determining the appropriate level of working capital for a firm requires (E)
a. Evaluating the risks associated with various levels of fixed assets and the types of debt used to finance these assets.
b. Changing the capital structure and dividend policy for the firm.
c. Maintaining short-term debt at the lowest possible level because it is ordinarily more expensive than long term debt.
d. Offsetting the profitability of current assets and current liabilities against the probability of technical insolvency.
e. Maintaining a high proportion of liquid assets to total assets in order to maximize the return on total investments. CMA
0689 1-10, 0696 1-16, RPCPA 0596
7
. Which of the following statements about current asset management is most correct? (E)
a. A positive net float means that a company has more cash available for its use than the amount shown in the company’s
books.
b. Use of a lockbox reduces the possibility that petty cash will be lost.
c. Depreciation has an impact on the cash budget.
d. Statements a and c are correct. Brigham

Working Capital
Working Capital Defined
*. The amount of long-term capital that is made to revolve in conducting operations and serves as the lifeblood of the company (E)
a. Paid-up capital c. Working capital
b. Net worth d. None of these RPCPA 0588
8
. The working capital is the difference between
A. Current assets and current liabilities. C. Total assets and total liabilities.
B. Fixed assets and fixed liabilities. D. Equity and cash. CMA 0692 1-25

3. Net Working Capital is the:


A. Difference between short-term assets and short term liabilities
B. Difference between long-term assets and long term liabilities
C. Difference between long-term assets and short term liabilities
D. None of the above B&M

Effect of Transactions on Working Capital


9
. Starrs Company has current assets of $300,000 and current liabilities of $200,000. Starrs could increase its working capital by the
(E)
A. Prepayment of $50,000 of next year's rent.
B. Refinancing of $50,000 of short-term debt with long-term debt.
C. Purchase of $50,000 of temporary investments for cash.
D. Collection of $50,000 of accounts receivable. CMA 1293 1-19
10
. Starrs has current assets of 300,000 and current liabilities of 200,000. Starrs could increase its working capital by the
A. Prepayment of 50,000 of next year's rent.
B. Refinancing of 50,000 of short-term debt with long-term debt.
C. Acquisition of land valued at 50,000 through the issuance of common shares.
D. Purchase of 50,000 of financial assets held for trading for cash. CMA 1293 1-19

*. Which of the following transactions causes an increase in working capital?


a. Sale of merchandise on credit at a price above cost.
b. Sale of marketable securities at a price below cost.
c. Collection of an account receivable.
d. Return to supplier of defective merchandise purchased on credit. Full credit allowed by supplier. RPCPA 1075

. Other things held constant, which of the following will cause an increase in working capital? (M)
a. Cash is used to buy marketable securities.
b. A cash dividend is declared and paid.
c. Merchandise is sold at a profit, but the sale is on credit.
d. Long-term bonds are retired with the proceeds of a preferred stock issue.
e. Missing inventory is written off against retained earnings. Brigham
11
. Which one of the following would increase the working capital of a firm?
A. Cash payment of payroll taxes payable.
B. Purchase of a new plant financed by a 20-year mortgage.
C. Cash collection of accounts receivable.
D. Refinancing a short-term note payable with a two-year note payable. CMA 1294 1-15
12
. If a firm increases its cash balance by issuing additional shares of common stock, working capital (E)
a. Remains unchanged and the current ratio remains unchanged.
b. Increases and the current ratio remains unchanged.
c. Increases and the current ratio decreases.
d. Increases and the current ratio increases. CMA 1294 1-30
13
. The following transactions occurred during a company's first year of operations:
I. Purchased a delivery van for cash
I. Borrowed money by issuance of short-term debt
III. Purchased treasury stock
Which of the items above caused a change in the amount of working capital?
A. I only. C. II and III only.
B. I and II only. D. I and III only. CIA 0593 IV-28

Comprehensive
14
. All of the following statements in regard to working capital are correct except (M)
a. Current liabilities are an important source of financing for many small firms.
b. Profitability varies inversely wit liquidity.
c. The hedging approach to financing involves matching maturities of debt with specific financing needs.
d. Financing permanent inventory buildup with long-term debt is an example of an aggressive working capital policy. CMA
0696 1-29

CASH MANAGEMENT
Function
15
. When managing cash and short-term investments, a corporate treasurer is primarily concerned with (E)
a. Maximizing rate of return.
b. Minimizing taxes.
c. Investing in Treasury bonds since they have no default risk.
d. Investing in common stock due to the dividend exclusion for federal income tax purposes.
e. Liquidity and safety. CMA 1295 I-12

1. Determining the appropriate target cash balance involves assessing the trade-off between:
A. Income and diversification
B. The benefit and cost of liquidity
C. Balance sheet strength and transaction needs
D. All of the above B&M

*. Which of the following is not a major function in cash management? (E)


a. Cash flow control c. Maximizing sales
b. Cash surplus investment d. Obtaining financing services RPCPA 0590

2. Firms would need to hold zero cash when:


A. Transaction-related needs are greater than cash inflows
B. Transaction-related needs are less than cash inflows
C. Transaction-related needs are not perfectly synchronized with cash inflows
D. Transaction-related needs are perfectly synchronized with cash inflows B&M
16
. Which of the following statements is most correct? (E)
a. A good cash management system would minimize disbursement float and maximize collections float.
b. If a firm begins to use a well-designed lockbox system, this will reduce its customers’ net float.
c. In the early 1980s, the prime interest rate hit a high of 21 percent. In 2000 the prime rate was considerably lower. That sharp
interest rate decline has increased firms’ concerns about the efficiency of their cash management programs.
d. If a firm can get its customers to permit it to pay by wire transfers rather than having to write checks, this will increase its net
float and thus reduce its required cash balances.
e. A firm that has such an efficient cash management system that it has positive net float can have a negative checkbook
balance at most times and still not have its checks bounce. Brigham

Motives for Holding Cash


17
. According to John Maynard Keynes, the three major motives for holding cash are for
A. Transactional, psychological, and social purposes.
B. Speculative, fiduciary, and transactional purposes.
C. Speculative, social, and precautionary purposes.
D. Transactional, precautionary, and speculative purposes. CMA 1286 1-32

*. A precautionary motive for holding excess cash is (E)


a. To enable a company to meet the cash demands from the normal flow of business activity.
b. To enable a company to avail itself of a special inventory purchase before prices rise to higher levels.
c. To enable a company to have cash to meet emergencies that may arise periodically.
d. To avoid having to use the various types of lending arrangements available to cover projected cash deficits. RPCPA 0595
18
. The amount of cash that a firm keeps on hand in order to take advantage of any bargain purchases that may arise is referred to as
its
A. Transactions balance. C. Precautionary balance.
B. Compensating balance. D. Speculative balance. CIA 1194 IV-15

25. Which of the following statements concerning zero balance accounts is not correct?
A. They are set up to handle disbursement activity
B. The account has a minimum amount at all times
C. Checks are automatically transferred into the account as checks presented for payment
D. The transfer is automatic and involves an accounting entry only
E. The master and the zero balance account locate at the same bank B&M

26. Which of the following is used to control disbursements?


A. Concentration banking C. Lock-box system
B. Zero-balance account D. Fedwire B&M
19
. All of the following are valid reasons for a business to hold cash and marketable securities except to
A. Satisfy compensating balance requirements.
B. Maintain adequate cash needed for transactions.
C. Meet future needs.
D. Earn maximum returns on investment assets. CMA 0694 1-22

. Which of the following statements is most correct? (M)


a. The cash balances of most firms consist of transactions, compensating, precautionary, and speculative balances. The total
desired cash balance can be determined by calculating the amount needed for each purpose and then summing them
together.
b. The easier a firm’s access to borrowed funds the higher its precautionary balances will be, in order to protect against sudden
increases in interest rates.
c. For some firms, holding highly liquid marketable securities is a substitute for holding cash because the marketable securities
accomplish the same objective as cash. Brigham
d. Firms today are more likely to rely on cash than on reserve borrowing power or marketable securities for speculative purposes
because of the need to move quickly.

12. A large firm may hold substantial cash balances because:


A. These balances are required by the bank
B. The company may have accounts in many different banks
C. The company may have a very decentralized organization
D. All of the above B&M

13. Most large firms hold a cash balance greater than most models imply because:
A. It is too difficult to estimate the costs of security transactions
B. Banks are compensated by account balances for payment of services
C. Corporations have few bank accounts and it is difficult to manage their cash
D. Cash is costless and need not be managed closely B&M

14. We should expect cash balances to increase when:


A. The transaction costs of buying or selling interest-bearing securities increase
B. Interest rates increase
C. Sales volume falls
D. Uncertainty about day-to-day or week-to-week cash flows decreases B&M

Methods of Accelerating Cash Collections


32. Banks generally offer the following cash management services:
A. Processing checks C. Running lock-boxes
B. Transferring funds D. All of the above B&M

33. Banks generally offer the following services:


A. Processing checks C. Providing advice and references
B. Running lock-boxes D. All of the above B&M

Disbursement Float
15. Checks written by the firm are said to generate:
A. Availability float C. Disbursement float
B. Ledger float D. Book float B&M

16. The difference between bank cash and book cash is called:
A. Disbursement float C. Availability float
B. Net float D. None of the above B&M
Maximizing Net Float
20
. Which of the following statements is most correct? (E)
a. A cash management system that minimizes collections float and maximizes disbursement float is better than one with higher
collections float and lower disbursement float.
b. A cash management system that maximizes collections float and minimizes disbursement float is better than one with lower
collections float and higher disbursement float.
c. The use of a lockbox is designed to minimize cash theft losses. If the cost of the lockbox is less than theft losses saved, then
the lockbox should be installed. Brigham
d. Other things held constant, a firm will need a smaller line of credit if it can arrange to pay its bills by the 5th of each month than
if its bills come due uniformly during the month.

. Which of the following statements is most correct? (M)


a. Poor synchronization of cash flows that results in high cash management costs can be partially offset by increasing
disbursement float and decreasing collections float.
b. The size of a firm’s net float is primarily a function of its natural cash flow synchronization and how it clears its checks.
c. Lockbox systems are used mainly for security purposes as well as to decrease the firm’s net float.
d. If a firm can speed up its collections and slow down its disbursements, it will be able to reduce its net float.
e. A firm practicing good cash management and making use of positive net float will bring its check book balance as close to
zero as possible, but must never generate a negative book balance. Brigham

Lock-Box System
21. The most common cash management technique used to speed up collections is:
A. Concentration banking C. Lock-boxes
B. Wire transfers D. In-house processing B&M

23. By getting closer to the source of payment, lock-boxes can be used to reduce:
A. Availability or clearing float C. In-house processing float
B. Mail float D. Disbursement float B&M
21
. A lock-box system
A. Reduces the need for compensating balances.
B. Provides security for late night deposits.
C. Reduces the risk of having checks lost in the mail.
D. Accelerates the inflow of funds. CMA 1293 1-20

. A lockbox plan is most beneficial to firms that (M)


a. Send payables over a wide geographic area.
b. Have widely disbursed manufacturing facilities.
c. Have a large marketable securities account to protect.
d. Hold inventories at many different sites.
e. Make collections over a wide geographic area. Brigham

Draft
22
. A working capital technique that delays the outflow of cash is (E)
A. Factoring. C. A lock-box system.
B. A draft. D. Electronic funds transfer. CMA 1293 1-21
23
. A working capital technique that increases the payable float and therefore delays the outflow of cash is (E)
A. Concentration banking. C. Electronic Data Interchange (EDI).
B. A draft. D. A lockbox system. CMA 1296 1-5

Automated Clearing House (ACH) Electronic Transfer


27. Electronic cash transfers offer several advantages, including:
A. A low marginal transactions cost C. Easy automation of record-keeping
B. A reduced float D. All of the above B&M

31. A check processed through ACH (Automated Clearing House):


A. Will clear immediately C. Will take two or three days to clear
B. Will clear the same day D. None of the above B&M
24
. An automated clearing house (ACH) electronic transfer is a(n)
a. Electronic payment to a company’s account at a concentration bank.
b. Check that must be immediately cleared by the Federal Reserve Bank.
c. Computer-generated deposit ticket verifying deposit of funds. CMA 0694 1-23)
d. Check-like instrument drawn against the payor and not against the bank.
*. The following practices will impact the cash flow of the company: (E)
a. Sales personnel are unequivocally responsible for collecting their credit sales.
b. Sales commissions are based on collected invoices.
c. Statement of accounts receivable are reconciled with customers and regularly sent for confirmation. RPCPA 0594
d. Automatic transfer of funds is arranged with banks regarding deposits of branches.

Wire Transfer
28. The fastest but most expensive way to transfer surplus funds from the local deposit bank to the concentration bank is:
A. A lock-box system D. An in-house processing float system
B. A mail float system E. An availability float system
C. A wire transfer B&M

Concentration Banking
22. Which of the following is a way for companies to speed up collections?
A. Remote disbursing C. Baumol model
B. Concentration banking D. All of the above B&M

Comprehensive
*. Which of the following actions would not be consistent with good management? (M)
a. Increased synchronization of cash flows.
b. Minimize the use of float.
c. Maintaining an average cash balance equal to that required as a compensating balance or that which minimizes total cost.
d. Use of checks and drafts in disbursing funds. RPCPA 0595

*. A company’s management is concerned about the large bank overdraft, which it wishes to reduce over the budget period of one
year. Which one of the four items below will not necessarily result in a lower bank overdraft? (E)
a. Reducing bad debts.
b. Taking longer credit from suppliers without any loss of discounts.
c. Reducing wastage and loss through damage of regularly-used stock items.
d. Reducing unit costs of production. RPCPA 0592
25
. Methods of accelerating cash collections include all of the following except (E)
A. Decentralized collections. C. Compensating balances.
B. Electronic funds transfers. D. Lockbox systems. CMA 0689 1-12
26
. A typical firm doing business nationally cannot expect to accelerate its cash inflow by
A. Establishing multiple collection centers throughout the country.
B. Employing a lockbox arrangement.
C. Initiating controls to accelerate the deposit and collection of large checks. CMA 1283 1-23
D. Maintaining compensating balances rather than paying cash for bank services.

Baumol’s Model of Cash Balances


3. Baumol's model of cash balances states that: Q = [(2 x T x C./ i]^0.5 where T = annual cash disbursement, C = cost per sale of T-
bills, i = interest rate. What is Q?
A. The number of times per annum bill should be sold
B. The average holding of bills
C. The amount of T-bills that should be sold at any one time
D. The minimum holding of cash B&M

9. The Baumol model determines the optimal cash balance by:


A. Balancing total costs against opportunity costs
B. Minimizing total costs of holding cash against trading securities costs
C. Balancing trading securities costs against total costs
D. Minimizing total costs less trading costs B&M

10. Concerning the Baumol model, which of the following is not correct (all other things equal)?
A. The optimum cash balance is higher at higher interest rates
B. The optimum cash balance is higher at higher fixed order costs
C. The optimum cash balance is higher at higher total cash requirement
D. All of the above are correct B&M

11. The Baumol cash balance model is limited by:


A. Assuming the cash flows are variable across the period
B. A smooth disbursement rate and now cash inflows over the period
C. Having a safety stock set to zero
D. Both B and C B&M
Cash Conversion Cycle
27
. An enterprise plans to produce a new product, which will typically be sold to other firms on credit. The cash conversion cycle
resulting from this new product can be measured as the length of time from
A. Cash purchases of raw materials to the collection of accounts receivable.
B. Cash purchases of raw materials to the time the final product is completed.
C. Cash purchases of raw materials to the sale of the product.
D. When the product is completed to the sale of the product. CIA 1193 IV-52
28
. Helena Furnishings wants to sharply reduce its cash conversion cycle. Which of the following steps would reduce its cash
conversion cycle? (E)
a. The company increases its average inventory without increasing its sales.
b. The company reduces its DSO.
c. The company starts paying its bills sooner, which reduces its average accounts payable without reducing its sales.
d. Statements a and b are correct. Brigham

. Ignoring cost and other effects on the firm, which of the following measures would tend to reduce the cash conversion cycle? (M)
a. Maintain the level of receivables as sales decrease.
b. Buy more raw materials to take advantage of price breaks.
c. Take discounts when offered.
d. Forgo discounts that are currently being taken.
e. Offer a longer deferral period to customers. Brigham
29
. Which of the following actions are likely to reduce the length of a company’s cash conversion cycle? (M)
a. Adopting a just-in-time inventory system that reduces the inventory conversion period.
b. Reducing the average days sales outstanding (DSO) on its accounts receivable.
c. Reducing the amount of time the company takes to pay its suppliers.
d. Statements a and b are correct. Brigham

Optimal Mix Between Cash and Short-term Investments


30
. When managing cash and short-term investments, a corporate treasurer is primarily concerned with (E)
A. Maximizing rate of return.
B. Minimizing taxes.
C. Investing in Treasury bonds since they have no default risk.
D. Liquidity and safety. CMA 1295 1-12
31
. When a company is evaluating whether the ratio of cash and marketable securities to total assets should be high or low, its
decision will be based upon
A. Financial leverage considerations.
B. Operating leverage considerations.
C. Risk-profitability trade-off considerations.
D. Flotation cost considerations. CMA 0684 1-5
32
. Some managers express the opinion that “cash management problems are nothing more than inventory problems.” They then
proceed to use cash management module, such as the EOQ model to determine the
a. Credit and collection policies.
b. Marketable securities level.
c. Proper relationship between current assets and current liabilities.
d. Proper blend of marketable securities and cash. CMA 1286 1-33
33
. Determining the amount and timing of conversions of marketable securities to cash is a critical element of a financial manager's
performance. In terms of the rate of return forgone on converted securities and the cost of such transactions, the optimal amount of
cash to be raised by selling securities is
A. Inversely related to the rate of return forgone and directly related to the cost of the transaction.
B. Directly related to the rate of return forgone and directly related to the cost of the transaction.
C. Directly related to the rate of return forgone and inversely related to the cost of the transaction.
D. Inversely related to the rate of return forgone and inversely related to the cost of the transaction. CIA 0593 IV-52
34
. The economic order quantity (EOQ) formula can be adapted in order for a firm to determine the optimal mix between cash and
marketable securities. The EOQ model assumes all of the following except
a. The cost of a transaction is independent of the dollar amount of the transaction and interest rates are constant over the short
run.
b. An opportunity cost is associated with holding cash, beginning with the first dollar.
c. The total demand for cash is known with certainty.
d. Cash flow requirements are random. CMA 0689 1-15

Comprehensive
*. The following are desirable in cash management except: (E)
a. Cash is collected at the earliest time possible.
b. Most sales are on cash basis and receivables are aged “current”
c. Post-dated checks are not deposited on time upon maturity.
d. All sales are properly receipted and promptly deposited intact. RPCPA 0593

*. In cash management, which of the following statements is false? (M)


a. Capital costs, delinquency costs, and default costs are costs associated with cash management.
b. Short costs, long costs, and procurement costs are costs associated with optimal cash balance model approach
c. Obtaining financing services and controlling cash flow are some of the major functions of cash management.
d. Funds sourcing and custodianship must be done at the lowest possible cost, where excess funds must be invested for a return
that is best in the market. RPCPA 1090

MARKETABLE SECURITIES
Criteria
*. The criteria that should be considered in investing surplus cash (E)
a. Safety of the company’s funds c. Yield on the principal
b. Liquidity of the principal d. All of the above RPCPA 0588
35
. Which of the following are criteria for selection among securities available for a marketable securities portfolio? (M)
A. Default risk, taxability, and relative yields.
B. Planning, major investment decisions, and interaction with capital markets.
C. The operating, payment, and cash conversion cycles. CIA 0589 IV-52
D. Inventory conversion, receivables conversion, and payable deferral periods.

. Which of the following is not a situation that might lead a firm to hold marketable securities? (M)
a. The firm has purchased a fixed asset that will require a large write-off of depreciable expense.
b. The firm must meet a known financial commitment, such as financing an ongoing construction project.
c. The firm must finance seasonal operations.
d. The firm has just sold long-term securities and has not yet invested the proceeds in earning assets.Brigham

Money Market Instruments


7. The following are money market instruments except:
A. T-bills C. Commercial paper
B. Federal agency discount notes D. Preferred stocks B&M

8. The three money market securities with the greatest volume of business are:
A. Treasury bills, commercial paper, repurchase agreements
B. Negotiable CDs, federal agency discount notes, T-bills
C. Commercial paper, bankers' acceptances, tax-exempt municipal notes
D. Federal agency discount notes, repurchase agreements, medium-term notes B&M

Marketable Securities Portfolio


. Which of the following statement completions is most correct? If the yield curve is upward sloping, then a firm’s marketable
securities portfolio, assumed to be held for liquidity purposes, should be (M)
a. Weighted toward long-term securities because they pay higher rates.
b. Weighted toward short-term securities because they pay higher rates.
c. Weighted toward U. S. Treasury securities to avoid interest rate risk.
d. Weighted toward short-term securities to avoid interest rate risk.
e. Balanced between long- and short-term securities to minimize the effects of either an upward or a downward trend in interest
rates. Brigham

Treasury Bills
36
. Which security is most often held as a substitute for cash? (E)
A. Treasury bills. C. Gold.
B. Common stock. D. Aaa corporate bonds. CMA 1289 1-14

9. For which of the following investments is there a very active secondary market?
A. Medium-term notes C. US Treasury bills
B. Commercial paper D. Repurchase agreements B&M
37
. The marketable securities with the least amount of default risk are
A. Federal government agency securities. C. Repurchase agreements.
B. U.S. Treasury securities. D. Commercial paper. CMA 0691 1-11
38
Of the following four investments, is considered to be the safest. (E)
a. commercial paper d. Treasury bonds
b. corporate bonds e. U. S. Agency issues
c. Treasury bills
39
. Which of the following financial instruments can be traded in international money markets?
A. Mortgages. C. U.S. Treasury bills.
B. Preferred shares. D. U.S. Treasury bonds. CIA 1195 IV-65

Repurchase Agreement
15. A repurchase agreement occurs when:
A. A company agrees to buy back its commercial paper before maturity
B. A bank depositor agrees, in advance, to re-invest money in a negotiable certificate of deposit
C. An investor buys part of a government security dealer's inventory and simultaneously agrees to sell it back
D. The federal government agrees to buy T-bills B&M

Negotiable Certificates of Deposit


40
. Which one of the following is not a characteristic of a negotiable certificate of deposit? Negotiable certificates of deposit (D)
A. Have a secondary market for investors.
B. Are regulated by the Federal Reserve System.
C. Are usually sold in denominations of a minimum of $100,000. CMA 0691 1-12
D. Have yields considerably greater than bankers' acceptances and commercial paper.

10. Negotiable CDs are issued by:


A. US Government C. Banks
B. Federal agencies D. Corporations B&M

Commercial Paper
41
. The best example of a marketable security with minimal risk would be (D)
A. Municipal bonds.
B. The common stock of a Aaa rated company.
C. The commercial paper of a Aaa rated company.
D. Stock options of a Aaa rated company. CMA 0688 1-15

EuroCommercial Paper
14. Commercial papers sold in the international markets are called:
A. Negotiable commercial papers C. MTNs
B. EuroCommercial papers D. None of the above B&M

Floating Rate Preferred Stock


16. Floating-rate preferred stock offers competitive rates of return with traditional money-market instruments but:
A. Is not rated by Moody's or Standard & Poor's
B. Still provides the corporate investor with the tax exclusion on dividend income
C. Has a fixed rate of dividend income
D. Offers a highly competitive trading market B&M

17. A tax-paying corporation would prefer to invest short-term money in:


A. Preferred stock C. Common stock
B. Floating-rate preferred stock D. Long-term bonds B&M

21. Even though the dividend rate on a floating-rate preferred stock is floating to keep in line with interest rates, the instrument still
suffers from risk such as:
A. A thin market causing potential principal risk and liquidity concerns
B. The risk of downgrades from the narrow range of issuers
C. The impact of tax law changes, which may reduce the after-tax value of the instrument
D. All of the above B&M

Money Market Mutual Funds


42
. In smaller businesses in which the management of cash is but one of numerous functions performed by the treasurer, various cost
incentives and diversification arguments suggest that surplus cash should be invested in
A. Commercial paper. C. Money market mutual funds.
B. Bankers' acceptances. D. Corporate bonds. CMA 1280 1-1

Eurodollar Deposit
2. "Eurodollars" or "international dollars" are:
A. Dollar deposits in banks outside the US
B. Dollars deposited in the US by foreigners
C. Dollars held by foreign governments
D. None of the above B&M

44. US dollars deposited in a German bank are called:


A. Deutsche dollars C. Eurodollars
B. American depositing receipts D. None of the above B&M
43
. Of the following, a characteristic of Eurobonds is that they are
A. Always denominated in Eurodollars.
B. Always sold in some country other than the one in whose currency the bond is denominated.
C. Sold outside the country of the borrower but are denominated in the currency of the country in which the issue is sold.
D. Generally issued as registered bonds. CIA 1195 IV-66
44
. Which of the following statements does not properly describe a Eurodollar deposit?
A. Eurodollar deposits are U.S. dollar deposits in banks outside of the U.S.
B. Eurodollar deposits are outside the direct control of the U.S. monetary authorities.
C. Eurodollar deposit rates tend to be lower than domestic U.S. rates on equivalent instruments. CIA 0594 IV-58
D. Interest rates on Eurodollar deposits are tied to the London Interbank Offer Rate (LIBOR).
45
. Interest rates received by depositors on Eurodollar deposits tend to be higher than domestic U.S. rates on equivalent instruments
because
A. Borrowers pay higher rates than domestic U.S. rates on equivalent instruments.
B. The deposits involve different currencies.
C. Eurodollar deposits are for smaller amounts.
D. The Eurodollar market is outside the direct control of the U.S. monetary authorities and has lower costs. CIA 0595 IV-59

3. There is a difference between "Eurodollar" or "international dollar" rate and short-term interest rate in the US because:
A. Banks are not subject to reserve requirements on international dollars
B. Banks need not insure international dollar deposits with the FDIC
C. Both A and B
D. None of the above B&M

Variable Rate Demand Bond


11. A variable rate demand bond (VRDB):
A. Is a long-term security
B. Has interest payments linked to the level of short-term interest rates
C. May periodically be sold back to the issuer at face value
D. Is tax-exempt
E. All of the above B&M

Not a Proper Investment for Idle Cash


46
. All of the following are alternative marketable securities suitable for investment except
A. U.S. Treasury bills. C. Commercial paper.
B. Eurodollars. D. Convertible bonds. CMA 0694 1-25

*. Which of the following investments is not likely to be a proper investment for temporary idle cash? (E)
a. Initial public offering of an established profitable conglomerate.
b. Commercial paper.
c. Treasury bills.
d. Treasury bonds due within one year. RPCPA 0595

Short Selling
47
. The term “short selling” is the
a. Selling of a security that was purchased by borrowing money from a broker.
b. Selling of a security that is not owned by the seller.
c. Selling of all the shares you own in a company in anticipation that the price will decline dramatically. CMA 1294 1-16
d. Betting that a stock will increase by a certain amount within a given period of time.
Riding the Yield Curve
*. In the process of investing of surplus cash, the term “riding the yield curve” refers to (E)
a. Diversifying securities portfolio so that the firm has an equal balance of long-term versus short-term securities.
b. Swapping different maturities of similar quality debt securities in order to obtain higher yield.
c. Purchasing only the longest maturities for given rates of return.
d. Adherence to the liquidity preference theory of securities investment. RPCPA 0596

RECEIVABLES MANAGEMENT
. Which of the following statements is most correct? (M)
a. A firm that makes 90 percent of its sales on credit and 10 percent for cash is growing at a rate of 10 percent annually. If the
firm maintains stable growth it will also be able to maintain its accounts receivable at its current level, since the 10 percent
cash sales can be used to manage the 10 percent growth rate.
b. In managing a firm’s accounts receivable it is possible to increase credit sales per day yet still keep accounts receivable fairly
steady if the firm can shorten the length of its collection period.
c. If a firm has a large percentage of accounts over 30 days old, it is a sign that the firm’s receivables management needs to be
reviewed and improved.
d. Since receivables and payables both result from sales transactions, a firm with a high receivables-to-sales ratio should also
have a high payables-to-sales ratio. Brigham

Monitoring Receivables
. Analyzing days sales outstanding (DSO) and the aging schedule are two common methods for monitoring receivables. However,
they can provide erroneous signals to credit managers when (E)
a. Customers’ payments patterns are changing.
b. Sales fluctuate seasonally.
c. Some customers take the discount and others do not.
d. Sales are relatively constant, either seasonally or cyclically. Brigham

Days Sales Outstanding


48
. An enterprise's receivables collection period is equal to
A. The inventory conversion period.
B. The cash conversion cycle.
C. The day's sales outstanding.
D. The inventory divided by average daily sales. CIA 0597 IV-27
49
. The average collection period for a firm measures the number of days (E)
a. After a typical credit sale is made until the firm receives the payment.
b. It takes a typical check to “clear” through the banking system.
c. Beyond the end of the credit period before a typical customer payment is received.
d. Before a typical account becomes delinquent. CMA 1295 1-4

. Which of the following statements is most correct? (M)


a. If a firm’s volume of credit sales declines then its DSO will also decline.
b. If a firm changes its credit terms from 1/20, net 40 days, to 2/10, net 60 days, the impact on sales can’t be determined
because the increase in the discount is offset by the longer net terms, which tends to reduce sales.
c. The DSO of a firm with seasonal sales can vary. While the sales per day figure is usually based on the total annual sales, the
accounts receivable balance will be high or low depending on the season.
d. An aging schedule is used to determine what portion of customers pay cash and what portion buy on credit.
e. Aging schedules can be constructed from the summary data provided in the firm’s financial statements. Brigham

Aging of Accounts Receivable


50
. An aging of accounts receivable measures the
a. Ability of the firm to meet short-term obligations.
b. Average length of time that receivables have been outstanding. CMA 1290 1-22
c. Percentage of sales that have been collected after a given period of time.
d. Amount of receivables that have been outstanding for given lengths of time.

. Which of the following statements is most correct? (M)


a. Other things held constant, the higher a firm’s days sales outstanding (DSO), the better its credit department.
b. If a firm that sells on terms of net 30 changes its policy and begins offering all customers terms of 2/10, net 30, and if no
change in sales volume occurs, then the firm’s DSO will probably increase.
c. If a firm sells on terms of 2/10, net 30, and its DSO is 30 days, then its aging schedule would probably show some past due
accounts.
d. Statements a and c are correct. Brigham

Credit & Collection Policy


*. The goal of credit policy is to (E)
a. Extend credit to the point where marginal profits equal marginal costs.
b. Minimize bad debt losses.
c. Minimize collection expenses.
d. Maximize sales. RPCPA 0597

*. It is held that the level of accounts receivable that the firm has or holds reflects both the volume of a firm’s sales on account and a
firm’s credit policies. Which one of the following items is not considered as part of the firm’s credit policies? (M)
a. The minimum risk group to which credit should be extended.
b. The extent (in terms of money) to which a firm will go to collect an account.
c. The length of time for which credit is extended.
d. The size of the discount that will be offered. RPCPA 1095
51
. The one item listed below that would warrant the least amount of consideration in credit and collection policy decisions is the
A. Quality of accounts accepted. C. Cash discount given. CMA 0684 1-3
B. Quantity discount given. D. Level of collection expenditures.
52
. When a company analyzes credit applicants and increases the quality of the accounts rejected, the company is attempting to (M)
A. Maximize sales. C. Increase the average collection period.
B. Increase bad-debt losses. D. Maximize profits. CMA 1286 1-34

. If easing a firm’s credit policy lengthens the collection period and results in a worsening of the aging schedule, then why do firms
take such actions? (E)
a. It normally stimulates sales.
b. To meet competitive pressures.
c. To increase the firm’s deferral period for payables.
d. Statements a and b are correct. Brigham

. Which of the following statements is most correct? (M)


a. If credit sales as a percentage of a firm’s total sales increases, and the volume of credit sales also increases, then the firm’s
accounts receivable will automatically increase.
b. It is possible for a firm to overstate profits by offering very lenient credit terms that encourage additional sales to financially
“weak” firms. A major disadvantage of such a policy is that it is likely to increase uncollectible accounts.
c. A firm with excess production capacity and relatively low variable costs would not be inclined to extend more liberal credit
terms to its customers than a firm with similar costs that is operating close to capacity.
d. Firms use seasonal dating primarily to decrease their DSO.
e. Seasonal dating with terms 2/15, net 30 days, with April 1 dating, means that if the original sale took place on February 1st,
the customer can take the discount up until March 15th, but must pay the net invoice amount by April 1st. Brigham

Average Gross Receivables Balance


53
. Which of the following represents a firm’s average gross receivables balances?
I. Days’ sales in receivables x accounts receivable turnover.
II. Average daily sales x average collection period.
III. Net sales ÷average gross receivables. (E)
a. I only. c. II only.
b. I and II only. d. II and III only. CMA 1296 1-18

Receivable Turnover
54
. A high turnover of accounts receivable, which implies a very short days-sales outstanding, could indicate that the firm
A. Has a relaxed (lenient) credit policy.
B. Offers small discounts.
C. Uses a lockbox system, synchronizes cash flows, and has short credit terms.
D. Has an inefficient credit and collection department. Gleim
55
. A decrease in the firm's receivable turnover ratio means that (D)
A. It is collecting credit sales more quickly than before.
B. It is collecting credit sales more slowly than before.
C. Sales have gone up.
D. Inventories have gone up. Gleim
56
. Accounts receivable turnover will normally decrease as a result of (E)
a. The write-off of an uncollectible account (assume the use of the allowance for doubtful accounts method).
b. A significant sales volume decrease near the end of the accounting period.
c. An increase in cash sales in proportion to credit sales.
d. A change in credit policy to lengthen the period for cash discounts. CMA 0690 4-14

Credit Terms
57
. An organization would usually offer credit terms of 2/10, net 30 when
a. The organization can borrow funds at a rate exceeding the annual interest cost.
b. The orgnization can borrow funds at a rate less than the annual interest cost.
c. The cost of capital approaches the prime rate.
d. Most competitors are offering the same terms and the organization has a shortage of cash. CMA 0691 1-7
58
. Which one of the following statements is most correct if a seller extends credit to a purchaser for a period of time longer than the
purchaser’s operating cycle? The seller (E)
a. Will have a lower level of accounts receivable than those companies whose credit period is shorter than the purchaser’s
operating cycle.
b. Is, in effect, financing more than just the purchaser’s inventory needs.
c. Can be certain that the purchaser will be able to convert the inventory into cash before payment is due.
d. Has no need for a stated discount rate or credit period. CMA 1296 1-13

Change in Credit Policy


59
. An increase in sales resulting from an increased cash discount for prompt payment would be expected to cause (M)
A. An increase in the operating cycle.
B. An increase in the average collection period.
C. A decrease in the cash conversion cycle.
D. A decrease in purchase discounts taken. CMA 1285 1-6

*. If a firm had been extending trade credit on a 2/10, net/30 basis, what change would be expected on the balance sheet of its
customer if the firm went to a net cash 30 policy? (M)
a. Increased payables and increased bank loan.
b. Increased receivables.
c. Decreased receivables.
d. Decrease in cash. RPCPA 0596
*. The level of accounts receivable will most likely increase as (M)
a. Cash sales increase and number of says sales.
b. Credit limits are expanded, credit sales increase, and credit terms remain the same.
c. Credit limits are expanded, cash sales increase, and aging of the receivables is improving.
d. Cash sales increase, current receivables ratio to past due increases, credit limits remain the same. RPCPA 0594
60
. A change in credit policy has caused an increase in sales, an increase in discounts taken, a decrease in the amount of bad debts,
and a decrease in the investment in accounts receivable. Based upon this information, the company’s (E)
A. Average collection period has decreased.
B. Percentage discount offered has decreased.
C. Accounts receivable turnover has decreased.
D. Working capital has increased. CMA 1296 1-6
61
. A change in credit policy has caused an increase in sales, an increase in discounts taken, a reduction of the investment in
accounts receivable, and a reduction in the number of doubtful accounts. Based on this information, we know that: (E)
a. Net profit has increased.
b. The average collection period has decreased.
c. Gross profit has declined.
d. The size of the discount offered has decreased.
e. The bad debt loss percentage has increased. RPCPA 1095, CMA 1289 1-15

*. The credit and collection policy of Amargo Co. provides for the imposition of credit block when the credit line is exceeded and/or
the account is past due. During the month, because of the campaign to achieve volume targets, the general manager has waived
the credit block policy in a number of instances involving big volume accounts. The likely effect of this move is (M)
a. Deterioration of aging of receivables only.
b. Increase in the level of receivables only.
c. Deterioration of aging and increase in the level of receivables.
d. Decrease in collections during the month the move was done. RPCPA 1094
62
. The sales manager at Ryan Company feels confident that, if the credit policy at Ryan’s were changed, sales would increase and,
consequently, the company would utilize excess capacity. The two credit proposals being considered are as follows:
Proposal A Proposal B
Increase in sales $500,000 $600,000
Contribution margin 20% 20%
Bad debt percentage 5% 5%
Increase in operating profits $75,000 $90,000
Desired return on sales 15% 15%
Currently, payments terms are net 30. The proposed payment terms for Proposal A and Proposal B are net 45 and net 90,
respectively. An analysis to compare these two proposals for the change in credit policy would include all of the following factors
except the
a. Cost of funds for Ryan.
b. Current bad debt experience.
c. Impact on the current customer base of extending terms to only certain customers.
d. Bank loan covenants on days’ sales outstanding. CMA 0697 1-14

*. A strict credit and collection policy is in place in Star Co. As Finance Director you are asked to advise on the propriety of relaxing
the credit standards in view of stiff competition in the market. Your advise will be favorable if (E)
a. The competitor will do the same thing to prevent lost sales.
b. there is a decrease in the distribution level of your product, and a more aggressive stance in necessary to retain market share.
c. The projected margin from increased sales will exceed the cost of carrying the incremental receivables.
d. The account receivable level is improving, so the company can afford the carrying cost of receivables. RPCPA 0594
63
. A company serves as a distributor of products by ordering finished products once a quarter and using that inventory to
accommodate the demand over the quarter. If it plans to ease its credit policy for customers, the amount of products ordered for its
inventory every quarter will be
A. Increased to accommodate higher sales levels.
B. Reduced to offset the increased cost of carrying accounts receivable.
C. Unaffected if safety stock is part of the current quarterly order.
D. Unaffected if the JIT inventory control system is used. CIA 0593 IV-53

Assignment
35. In a loan arranged through the assignment of accounts receivable the lender:
A. Accepts the actual receivable to be collected
B. Has a lien on the receivables and recourse to the borrower
C. Assumes full risk of default
D. All of the above B&M
Factoring
64
. Factoring is the (E)
A. Selling of accounts receivable by one company to another.
B. Selling of inventory by one company to another.
C. Conversion of accounts receivable to bad debt on financial statements for accounts that are long overdue. CIA 0590 IV-50
D. Adjustment of inventories on financial statements for supplies that have become obsolete.

Pledging
65
. Short-term borrowing using accounts receivable and characterized by the fact that the lender not only has a claim against the
receivables but also has recourse against the borrower is
A. Factoring. C. Selling.
B. Pledging. D. Hedging. CIA 0589 IV-53

SHORT-TERM FINANCING
Short-term & Long-term Financing
*. Debt capital be it long-term or short-term in nature, can be raised from a number of different sources including
a. Trade credit c. All of the above
b. Bank loan d. None of the above RPCPA 1091
66
. In general, as a company increases the amount of short-term financing relative to long-term financing, the
A. Greater the risk that it will be unable to meet principal and interest payments.
B. Leverage of the firm increases.
C. Likelihood of having idle liquid assets increases.
D. Current ratio increases. CMA 0688 1-17

. Firms generally choose to finance temporary assets with short-term debt because (M)
a. Matching the maturities of assets and liabilities reduces risk.
b. Short-term interest rates have traditionally been more stable than long-term interest rates.
c. A firm that borrows heavily long-term is more apt to be unable to repay the debt than a firm that borrows heavily short-term.
d. The yield curve has traditionally been downward sloping. Brigham
e. Sales remain constant over the year, and financing requirements also remain constant.
67
. Which of the following statements is most correct? (M)
a. Under normal conditions, a firm’s expected ROE would probably be higher if it financed with short-term rather than with long-
term debt, but the use of short-term debt would probably increase the firm’s risk.
b. Conservative firms generally use no short-term debt and thus have zero current liabilities.
c. A short-term loan can usually be obtained more quickly than a long-term loan, but the cost of short-term debt is likely to be
higher than that of long-term debt.
d. If a firm that can borrow from its bank buys on terms of 2/10, net 30, and if it must pay by Day 30 or else be cut off, then we
would expect to see zero accounts payable on its balance sheet.
e. If one of your firm’s customers is “stretching” its accounts payable, this may be a nuisance but does not represent a real
financial cost to your firm as long as the firm periodically pays off its entire balance.Brigham

. Which of the following statements is most correct? (M)


a. Under normal conditions the shape of the yield curve implies that the interest cost of short-term debt is greater than that of
long-term debt, although short-term debt has other advantages that make it desirable as a financing source.
b. Flexibility is an advantage of short-term credit but this is somewhat offset by the higher flotation costs associated with the
need to repeatedly renew short-term credit.
c. A short-term loan can usually be obtained more quickly than a long-term loan but the penalty for early repayment of a short-
term loan is significantly higher than for a long-term loan.
d. Statements about the flexibility, cost, and riskiness of short-term versus long-term credit are dependent on the type of credit
that is actually used. Brigham
e. Short-term debt is often less costly than long-term debt and the major reason for this is that short-term debt exposes the
borrowing firm to much less risk than long-term debt.

Trade Credit
68
. Which one of the following provides a spontaneous source of financing. (E)
a. Accounts payable. c. Accounts receivable.
b. Mortgage bonds. d. Debentures. CMA 1295 1-15
69
. Which one of the following is a spontaneous source of financing?
A. Notes payable. C. Prepaid interest.
B. Long-term debt. D. Trade credit. CMA 1289 1-20
70
. Which one of the following financial instruments generally provides the largest source of short-term credit for small firms?
A. Installment loans. C. Trade credit.
B. Commercial paper. D. Mortgage bonds. CMA 1295 1-9
71
. Which one of the following statements about trade credit is correct?
a. Not an important source of financing for small firms.
b. A source of long-term financing to the seller.
c. Subject to risk of buyer default.
d. Usually an inexpensive source of external financing. CMA 1296 1-12
72
. The correct equation for calculating the approximate percentage cost, on an annual basis, of not taking trade discounts is CIA
1195 IV-53
A. Discount % 360
x
100 - Discount % Days credit is outstanding - Discount period
B. Discount % 360
x
100 Days credit is outstanding - Discount period
C. 100− Discount % 360
x
Discount % Days credit is outstanding - Discount period
D. Discount % Days credit is outstanding - Discount period
x
100 - Discount % 360
73
. A company obtaining short-term financing with trade credit will pay a higher percentage financing cost, everything else being
equal, when (D)
A. The discount percentage is lower.
B. The items purchased have a higher price.
C. The items purchased have a lower price.
D. The supplier offers a longer discount period. CIA 0597 IV-51
74
. Which one of the following statements concerning cash discount is correct? (E)
A. The cost of not taking a 2/10, net 30 cash discount is usually less than the prime rate.
B. With trade terms of 2/15, net 60, if the discount is not taken, the buyer receives 45 days of free credit. CMA 1295 1-7
C. The cost of not taking the discount is higher for terms of 2/10, net 60 than for 2/10, net 30.
D. The cost of not taking a cash discount is generally higher than the cost of a bank loan.
75
. A small retail business would most likely finance its merchandise inventory with
A. Commercial paper. C. A line of credit.
B. A terminal warehouse receipt loan. D. A chattel mortgage. CMA 0689 1-17
76
. Which of the following statements is most correct? (M)
a. Accruals are an expensive way to finance working capital.
b. A conservative financing policy is one in which the firm finances all of its fixed assets with long-term capital and part of its
permanent current assets with short-term, nonspontaneous credit.
c. If a company receives trade credit under the terms 2/10 net 30, this implies the company has 10 days of free trade credit.
d. Statements a and b are correct. Brigham
77
. Merkle, Inc. has a temporary need for funds. Management is trying to decide between not taking discounts from one of their three
biggest suppliers, or a 14.75% per annum renewable discount loan from its bank for 3 months. The suppliers' terms are as follows:
Fort Co. 1/10, net 30
Riley Manufacturing Co. 2/15, net 60
Shad, Inc. 3/15, net 90
Using a 360-day year, the cheapest source of short-term financing in this situation is (D)
A. The bank. C. Riley Manufacturing Co.
B. Fort Co. D. Shad, Inc. CMA 1283 1-25

Bank Loans
. Which one of the following aspects of banks is considered most relevant to businesses when choosing a bank? (M)
a. Convenience of location.
b. Competitive cost of services provided.
c. Size of the bank’s deposits.
d. Experience of personnel.
e. Loyalty and willingness to assume lending risks. Brigham

22. Which of the following describes short-term bank loans?


A. Often secured by a factoring agent
B. Almost never secured by inventory
C. Often prearranged as a line of credit
D. Cannot be from an international bank because of Federal Reserve System regulations
E. None of the above B&M
23. Which of the following describes short-term bank loans?
A. If unsecured, banks often require borrower to "clean up" the loan for 1 month in the year
B. Often secured by commercial paper
C. Almost never secured by accounts receivable
D. Cannot be from an international bank because Federal Reserve System regulations prohibit Eurodollar borrowing
E. None of the above B&M

Term Loan
24. Which of the following statements describes bank term loans?
A. Typical maturity is between 2 and 4 years
B. Usually repaid in level amounts over the term of the loan
C. Interest rate is usually fixed
D. Borrower is often obliged to have the bank mange its corporate cash accounts B&M

25. Which of the following statements describe bank term loans?


A. Typical maturity is between 1 and 2 years
B. Usually repaid in increasing amounts over the term of the loan
C. Interest rate is usually fixed
D. Borrower is often obliged to maintain compensating balance B&M

Interest rates
78
. Short-term interest rates are
a. Usually lower than long-term rates.
b. Usually higher than long-term rates.
c. Lower than long-term rates during periods of high inflation only.
d. Not significantly related to long-term rates. CMA 0691 1-5
79
. The prime rate is the (D)
A. Size of the commitment fee on a commercial bank loan.
B. Effective cost of a commercial bank loan.
C. Effective cost of commercial paper.
D. Rate charged on business loans to borrowers with high credit ratings. CMA 0688 1-18
80
. The prime lending rate of commercial banks is an announced rate and is often understated from the viewpoint of even the most
credit-worthy firms. Which one of the following requirements always results in a higher effective interest rate?
A. A floating rate for the loan period.
B. A covenant that restricts the issuance of any new unsecured bonds during the existence of the loan.
C. The imposition of a compensating balance with an absolute minimum that cannot be met by current transaction balances.
D. The absence of a charge for any unused portion in the line of credit. CMA 1280 1-3

Nominal Interest Rate


81
. A one year, $20,000 loan with a 10% nominal interest rate provides the borrower with the use of <List A> if interest is charged on a
<List B> basis. (E)
CIA 0595 IV-50 A. B. C. D.
List A $18,000 $20,000 $20,000 $22,000
List B Simple Simple Discount Discount

Compensating Balance
82
. A minimum checking account balance that a firm must maintain with a commercial bank is a
A. Transactions balance. C. Precautionary balance.
B. Compensating balance. D. Speculative balance. CIA 1190 IV-49
83
. A compensating balance
A. Compensates a financial institution for services rendered by providing it with deposits of funds. CMA 0688 1-13
B. Is used to compensate for possible losses on a marketable securities portfolio.
C. Is a level of inventory held to compensate for variations in usage rate and lead time.
D. Is an amount paid by financial institutions to compensate large depositors.

. Which of the following statements is most correct? (M)


a. Compensating balance requirements apply only to businesses, not to individuals.
b. Compensating balances are essentially costless to most firms, because those firms would normally have such funds on hand
to meet transactions needs anyway.
c. If the required compensating balance is larger than the transactions balance the firm would ordinarily hold, then the effective
cost of any loan requiring such a balance is increased.
d. Banks are prohibited from earning interest on the funds they force businesses to keep as compensating balances. Brigham
Effective Rate
84
. Discounted interest is based on the borrowed amount but is paid in advance. The formula for calculating the discounted interest
rate for a 1-year loan is (E) Gleim
Interest Interest
A. Borrowed amount C. Borrowed amount - Interest
Interest
B. Average borrowed amount D. None of the answers are correct.

Collateral
*. In assessing the loan value of inventory, a banker will normally be concerned about the portion of inventory that is work-in-process
because (E)
a. WIP inventory is relatively easy to sell because it does not represent a raw material or a finished product.
b. WIP inventory usually has the highest loan value of the different inventory types.
c. WIP generally has the lowest marketability of the various types of inventories.
d. WIP represents a lower investment by a corporation as opposed to other types of inventories. RPCPA 0596

34. Firms provide the following as security for short-term loans:


A. Accounts receivables C. Securities
B. Inventories D. All of the above B&M

Syndicated Loan
30. When banks have to make large loans, they form a group of banks for the purpose of making the loan. The group is called a:
A. Bank holding company C. Golden umbrella
B. Syndicate D. Conglomerate B&M

Loan Sales by Commercial Banks


32. Loan sales by commercial banks may take the form of:
A. Loan assignments D. All of the above
B. Loan participations E. A and B
C. Loan syndications B&M

33. Loan participations are different from syndicated loans in that:


A. The lead bank provides a "certificate of participation" to each bank
B. Each participating bank has a separate loan agreement with the borrower
C. The loans are transferred to new lenders
D. None of the above B&M

Banker’s Acceptance
85
. The credit instrument known as a banker's acceptance
A. Calls for immediate payment upon delivery of the shipping documents to the bank's customer and acceptance of goods by the
bank.
B. Involves an invoice being signed by the banker upon receipt of goods, after which both the banker and the seller record the
transaction on their respective books.
C. Is a time draft payable on a specified date and guaranteed by the bank.
D. Is a method of sales financing in which the bank retains title to the goods until the buyer has completed payment. CIA
0596 IV-41

Unsecured Credit
86
. The following forms of short-term borrowing are available to a firm:
 Floating lien
 Factoring
 Revolving credit
 Chattel mortgages
 Bankers' acceptances
 Lines of credit
 Commercial paper
The forms of short-term borrowing that are unsecured credit are
A. Floating lien, revolving credit, chattel mortgage, and commercial paper.
B. Factoring, chattel mortgage, bankers' acceptances, and line of credit.
C. Floating lien, chattel mortgage, bankers' acceptances, and line of credit. CMA 1286 1-35
D. Revolving credit, bankers' acceptances, line of credit, and commercial paper.

Commercial Paper
87
. Short-term, unsecured promissory notes issued by large firms are known as
A. Agency securities. C. Commercial paper. CMA 0689 1-13
B. Bankers' acceptances. D. Repurchase agreements.
88
. Commercial paper
A. Has a maturity date greater than 1 year.
B. Is usually sold only through investment banking dealers.
C. Ordinarily does not have an active secondary market.
D. Has an interest rate lower than Treasury bills. CMA 0691 1-10
89
. The principal advantage of using commercial paper as a short-term financing instrument is that it (E)
A. Is usually cheaper than a commercial bank loan.
B. Is readily available to almost all companies.
C. Offers security, i.e., collateral, to the lender.
D. Can be purchased without commission costs. CMA 1289 1-22
90
. Which one of the following responses is not an advantage to a corporation that uses the commercial paper market for short-term
financing?
a. This market provides more funds at lower rates than other methods provides.
b. The borrower avoids the expense of maintaining a compensating balance with a commercial bank.
c. There are no restrictions as to the type of corporation that can enter into this market.
d. This market provides a broad distribution for borrowing. CMA 0696 1-14

37. Large companies often raise short-term debt by selling:


A. Bonds C. Medium term notes
B. Debentures D. Commercial paper B&M

. Which of the following statements concerning commercial paper is incorrect? (E)


a. Commercial paper is generally written for terms less than 270 days.
b. Commercial paper generally carries an interest rate below the prime rate.
c. Commercial paper is sold to money market mutual funds, as well as to other financial institutions and nonfinancial
corporations.
d. Commercial paper can be issued by virtually any firm so long as it is willing to pay the going interest rate. Brigham
e. Commercial paper is a type of unsecured promissory note issued by large, strong firms.
91
. Which of the following statements is incorrect? (E)
a. Commercial paper can be issued by virtually any firm so long as it is willing to pay the going interest rate.
b. Accruals represent a source of “free” financing in the sense that no explicit interest is paid on these funds.
c. A conservative approach to working capital will result in all permanent assets being financed using long-term securities.
d. The risk to the firm of borrowing with short-term credit is usually greater than with long-term debt. Added risk can stem from
greater variability of interest costs on short-term debt.
e. Trade credit is often the largest source of short-term credit. Brigham

Commercial Paper Rating


12. Which of the following is a commercial paper rating by Moody's?
A. Aaa C. P-1
B. Baa D. All of the above B&M

13. Which of the following is a commercial paper rating by Moody's?


A. P-2 C. Aaa
B. Aa D. None of the above B&M

Agency Securities
92
. Short-term securities issued by the Federal Housing Administration are known as
A. Agency securities. C. Commercial paper.
B. Bankers' acceptances. D. Repurchase agreements. CMA 0689 1-14

Secured Short-term Financing


93
. An example of secured short-term financing is
A. Commercial paper. C. A revolving credit agreement.
B. A warehouse receipt. D. Line of credit. CIA 1191 IV-56

Inventory Loans
36. The three basic forms of inventory loans include:
A. Blanket inventory lien, field warehouse financing, and line of credit
B. Blanket inventory lien, line of credit, and trust receipt
C. Blanket inventory lien, field warehouse financing, and trust receipt
D. Field warehouse financing, line of credit, and trust receipt B&M
Alternative Financing Arrangements
94
. A corporation is currently experiencing cash-flow problems and has determined that it is in need of short-term credit. It can either
use its trade credit on $100,000 of accounts payable with terms of 1/10, net 30 or a 30-day note with a 20% annual simple interest
rate. Which is the best alternative, and what is its effective rate of interest (rounded to a whole percentage and using a 360-day
year)? (M)
A. The trade credit. Its effective rate is 10%.
B. The trade credit. Its effective rate is 20%.
C. The note. Its effective rate is 17%.
D. The note. Its effective rate is 20%. CIA 1188 IV-54
95
. A short-term bank loan will have a higher effective financing cost if it has which combination of characteristics?
A. A 10% compensating balance and regular interest.
B. A 10% compensating balance and discount interest.
C. A 20% compensating balance and regular interest.
D. A 20% compensating balance and discount interest. CIA 1196 IV-52
96
. A manufacturing firm wants to obtain a short-term loan and has approached several lending institutions. All of the potential lenders
are offering the same nominal interest rate, but the terms of the loans vary. Which of the following combinations of loan terms will
be most attractive for the borrowing firm?
A. Simple interest, no compensating balance.
B. Discount interest, no compensating balance.
C. Simple interest, 20% compensating balance required.
D. Discount interest, 20% compensating balance required. CIA 0594 IV-51
97
. A company is arranging debt financing for the purchase of a new piece of equipment that has a 5-year expected useful life. Which
of the following alternative financing arrangements has the lowest effective annual percentage rate if each has a quoted nominal
rate of 9.5%?
A. A 5-year term loan with interest compounded annually.
B. A 10-year term loan with interest compounded semiannually.
C. A 5-year term loan with interest compounded quarterly.
D. A 10-year term loan with interest compounded monthly. CIA 1196 IV-45
98
. A company will receive cash from sales in 1 year that can be used to pay for materials. The supplier will allow payment in 1 year. If
the company pays the supplier immediately, it will receive a 20% discount off the $100,000 purchase price, but it must borrow the
full amount. A bank has offered the company three alternatives:
1. A 1-year loan at 18% with no other fees,
2. A 1-year loan at 15% with the provision that it maintains 20% of whatever amount it borrows as noninterest-bearing
compensating balances over the life of the loan, or
3. A guaranteed line of credit of $100,000 at 17% with the provision that the bank will collect a 1% fee on the average amount of
unused funds. The company expects to borrow no other funds.The company would achieve the lowest cost of financing by
(M)
A. Allowing the supplier to finance the materials and making payment at the end of 1 year.
B. Accepting the 1-year loan at 18% with no other provisions.
C. Accepting the 1-year loan at 15% with the compensating balance provisions.
D. Accepting the guaranteed line of credit at 17% with the fee required on the average amount of unused funds. CIA
1186 IV-43
ANSWER EXPALANATIONS
1
.REQUIRED: The true statement about a more conservative working capital policy.
DISCUSSION: (D) A conservative working capital policy results in an increase in working capital (current assets –
current liabilities). It is typified by a reduction in liquidity risk, increasing the current ratio, whether by decreasing current
liabilities or increasing current assets, minimizes the risk that the company will not be able to meet its obligations as they
fall due. Thus, an increasing ratio of current to noncurrent higher returns on long-term assets in order to guard against
short-term cash flow problems.
Answer (A) is incorrect because an increase in current liabilities relative to noncurrent liabilities would increase liquidity
risk. Answer (B) is incorrect because a decrease in the normal operating cycle permits a lower level of working capital.
If assets can be converted to cash more quickly, current assets can be reduced. Answer (C) is incorrect because a
decrease in the quick ratio signifies that quick assets ( cash, receivables, and marketable securities) are decreasing
relative to current liabilities.

2(F)(T) Interest
C+ =
k Borrowed amount - interest
2
.Answer (C) is correct. A conservative working capital policy minimizes liquidity risk by increasing working capital
(current assets - current liabilities). The result is that the company forgoes the potentially higher returns available from
using the additional working capital to acquire long-term assets. A conservative working capital policy is characterized
by a higher current ratio (current assets/current liabilities) and acid-test ratio (quick assets/current liabilities). Thus, the
company will increase current assets or decrease current liabilities. A conservative policy finances assets using long-
term or permanent funds rather than short-term sources.
Answer (A) is incorrect because a decrease in the acid-test ratio suggests an aggressive policy. A conservative
company wants a higher acid-test ratio, that is, more liquid assets relative to liabilities. Answer (B) is incorrect because a
conservative company wants working capital to be financed from long-term sources. Answer (D) is incorrect because a
conservative company seeks more liquid (marketable) investments.

2(F)(T) Interest
C+ =
k Borrowed amount - interest
3
.REQUIRED: The working capital financing policy that subects a firm to the greatest risk of being unable to meet
maturing obligations.
DISCUSSION: (C) Fluctuating curent assets can often be financed with short-term debt because the periodic liquidition
of the assets provides funds to pay off the debt. However, financing permanent current assets with short-term debt is a
risky strategy because the assets may not be liquidated in time to pay off the debt at maturity.
Answers (A), and (B) are incorrect because it is not particularly risky to finance working captial needs from long-term
debt sources. Answer (D) is incorrect because financing fluctuating current assets with short-term debt is not as risky as
financing permanent current assets with short-term debt.

2(F)(T) Interest
C+ =
k Borrowed amount - interest
4
.Answer (B) is correct. When a firm has an aggressive working capital policy, management keeps the investment in
working capital at a minimum. Thus, a growing company would want to invest its funds in capital goods and not in idle
assets. This policy maximizes return on investment at the price of the risk of minimal liquidity.
Answer (A) is incorrect because the growing firm is more apt to emphasize production rather than protecting against
technical insolvency by maintaining a high level of working capital. Answer (C) is incorrect because the company will
prefer to expend funds on capital goods. Answer (D) is incorrect because the company needs its profits to invest in new
production equipment in order to grow.

2(F)(T) Interest
C+ =
k Borrowed amount - interest
5
.Answer (B) is correct. A conservative working capital management financing policy uses permanent capital to finance
permanent asset requirements and also some or all of the firm's seasonal demands. Thus, Lott's current ratio (current
assets/current liabilities) will be high since its current liabilities will be relatively low. An aggressive policy entails
financing some fixed assets and all the current assets with short-term capital. This policy results in a lower current ratio.
Answer (A) is incorrect because Clay's aggressive policy would result in more short-term debt, with attendant renewal
problems and high risk. Lott's conservative policy would produce more long-term debt or equity financing. Answer (C) is
incorrect because Clay is subject to greater liquidity risk than Lott since it has greater short-term debt. Hence, it is at
greater risk of being unable to meet its maturing obligations. Answer (D) is incorrect because a more conservative
company would tend to finance by means of equity rather than debt capital. Thus, the more conservative company
would have less interest expense.

2(F)(T) Interest
C+ =
k Borrowed amount - interest
6
.REQUIRED: The true statement about the determination of the appropriate level of working capital.
DISCUSSION: (C) A company must maintain a level of working capital sufficient to pay bills as they come due. Failure
to do so is a technical insolvency and can result in involuntary bankruptcy. Unfortunately, holding current assets for
purposes of paying bills is not profitable for a company because they usually offer a low return compared with longer-
term investments. Thus, the skillful management of working capital requires a balancing of a firm’s desire for profit with
its need for adequate liquidity.
Answer (A) is incorrect because management of fixed assets is not a factor in working capital management. Answer (B)
is incorrect because capital structure and dividend policy are factors involved in capital structure finance, not in working
capital financial management. Answer (C) is incorrect because short-term debt is usually less expensive than long-term
debt. Answer (E) is incorrect because maintaining a high level of liquid assets is usually not profitable.

2(F)(T) Interest
C+ =
k Borrowed amount - interest
7
.Statements a and c are correct; therefore, statement d is the appropriate choice. A lockbox speeds collections of
receivables; it doesn’t ensure that petty cash will be safe. Although depreciation is a noncash expense, it does affect
taxes, which are a cash expense.

2(F)(T) Interest
C+ =
k Borrowed amount - interest
8
.REQUIRED: The definition of net working capital.
DISCUSSION: (A) Net working capital is defined as the difference between current assets and current liabilities.
Working capital is a measure of short-term solvency.
Answer (B) is incorrect because working capital refers to the difference between current assets and current liabilities;
fixed assets are not a component. Answer (C) is incorrect because total assets and total liabilities are not components
of working capital; only current items are included. Answer (D) is incorrect because shareholders’ equity is not a
component of working capital; only current items are included in the concept of working capital.

2(F)(T) Interest
C+ =
k Borrowed amount - interest
9
.Answer (B) is correct. Working capital is the excess of current assets over current liabilities. Refinancing a short-term
debt with a long-term debt decreases current liabilities, and the result is an increase in working capital.
Answer (A) is incorrect because a prepayment of expenses does not change current assets or current liabilities. Cash
decreases by the same amount that prepaid rent increases. Answer (C) is incorrect because the purchase of temporary
investments does not affect total current assets; cash is replaced by temporary investments, another current asset.
Answer (D) is incorrect because the collection of a receivable has no effect on total current assets. The receivable is
replaced by an equal amount of cash.
2(F)(T) Interest
C+ =
k Borrowed amount - interest
10
.Answer (B) is correct. Working capital is the excess of current assets over current liabilities. Refinancing a short-term
debt with a long-term debt decreases current liabilities, and the result is an increase in working capital.
Answer (A) is incorrect because a prepayment of expenses does not change current assets or current liabilities. Cash
decreases by the same amount that prepaid rent increases. Answer (C) is incorrect because the acquisition of land for
common shares does not affect either current assets or current liabilities. Answer (D) is incorrect because the purchase
of financial assets held for trading does not affect total current assets; cash is replaced by the financial assets held for
trading, another current asset.

2(F)(T) Interest
C+ =
k Borrowed amount - interest
11
.Answer (D) is correct. Working capital equals current assets minus current liabilities. Refinancing a short-term note with
a two-year note payable decreases current liabilities, thus increasing working capital.
Answer (A) is incorrect because a cash payment of payroll taxes decreases current assets and current liabilities by
equal amounts. Answer (B) is incorrect because buying a new plant with a 20-year mortgage has no effect on current
assets or current liabilities. Answer (C) is incorrect because cash collection of an account receivable increases one
current asset and decrease another by the same amount.

2(F)(T) Interest
C+ =
k Borrowed amount - interest
12
.REQUIRED: The effect on working capital and the current ratio of issuing common stock for cash.
DISCUSSION: (D) Working capital equals current assets minus current liabilities. The current ratio equals current
assets divided by current liabilities. Selling stock for cash increases current assets and stockholders’ equity, with no
effect on current liabilities. The result is an increase in working capital and the current ratio.
Answers (A), (B) and (C) are incorrect because both working capital and the current ratio increase.

2(F)(T) Interest
C+ =
k Borrowed amount - interest
13
.Answer (D) is correct. Working capital is computed by deducting total current liabilities from total current assets. The
purchase of a delivery van for cash reduces current assets and has no effect on current liabilities. The borrowing of cash
by incurring short-term debt increases current assets by the same amount as it increases current liabilities; hence, it will
have no effect on working capital. The purchase of treasury stock decreases current assets but has no effect on current
liabilities. Thus, the purchases of the van and treasury stock affect working capital.
Answer (A) is incorrect because the purchases of the van and treasury stock affect working capital. Answer (B) is
incorrect because the purchases of the van and treasury stock but not the issuance of short-term debt affect working
capital. Answer (C) is incorrect because the purchases of the van and treasury stock but not the issuance of short-term
debt affect working capital.

2(F)(T) Interest
C+ =
k Borrowed amount - interest
14
.REQUIRED: The false statement about working capital.
DISCUSSION: (D) Financing permanent inventory build-up which is essentially a long-term investment, with long-term
debt is a moderate or conservative working capital policy. An aggressive policy uses short-term relatively low-cost debt
to finance the inventory buildup. It focuses on high profitability potential, despite high risk and low liquidity. An
aggressive policy reduces the current ratio and accepts a higher risk of short-term lack of liquidity. Financing inventory
with long-term debt increases the current ratio and accepts higher borrowing costs in exchange for greater liquidity and
lower risk.
Answer (A) is incorrect because current liabilities, e.g., trade credit is a major source of funds for small firms. Answer
(B) is incorrect because liquid investments tend to have lower returns. Answer (C) is incorrect because matching of
asset and liability maturities is a moderate policy that minimizes risk. The expectation is that cash flows from the assets
will be available to meet obligations for the liabilities.

2(F)(T) Interest
C+ =
k Borrowed amount - interest
15
.REQUIRED: The primary concern when managing cash and short-term investments.
DISCUSSION: (E) Cash and short-term investments are crucial for a firm’s continuing success. Sufficient liquidity must
be available to meet payments as they come due. At the same time, liquid assets are subject to significant control risk.
Therefore, liquidity and safety are the primary concerns of the treasurer when dealing with a highly liquid assets. Cash
and short-term investments are held because of their ability to facilitate routine operations of the company. These
assets are not held for purposes of achieving investment returns.
Answer (A) is incorrect because most companies are not in business to earn high returns on liquid assets (i.e., they are
held to facilitate operations). Answer (B) is incorrect because the holding of cash and cash-like assets is not a major
factor in controlling taxes. Answer (C) is incorrect because investments in Treasury bonds do not have sufficient
liquidity to serve as short-term assets. Answer (D) is incorrect because common stocks do not have sufficient liquidity to
serve as short-term cash resources.

2(F)(T) Interest
C+ =
k Borrowed amount - interest
16
.A very efficient cash management system could allow a firm to operate with positive net float where the firm has a
negative checkbook balance at most times but still does not bounce its checks. The other statements are false. A good
cash management system maximizes disbursement float and minimizes collections float. A well-designed lockbox
system minimizes collections float which would increase a firm’s net float. Increases in interest rates raise the
opportunity cost of idle cash. A firm prefers to write checks, maximizing its disbursement float and increasing its net
float.

2(F)(T) Interest
C+ =
k Borrowed amount - interest
17
.Answer (D) is correct. John Maynard Keynes, founder of Keynesian economics, concluded that there were three major
motives for holding cash: for transactional purposes as a medium of exchange, precautionary purposes, and speculative
purposes (but only during deflationary periods).
Answer (A) is incorrect because the three major motives for holding cash, according to Keynesian economics, are for
transactional, precautionary, and speculative purposes. Answer (B) is incorrect because the three major motives for
holding cash, according to Keynesian economics, are for transactional, precautionary, and speculative purposes.
Answer (C) is incorrect because the three major motives for holding cash, according to Keynesian economics, are for
transactional, precautionary, and speculative purposes.

2(F)(T) Interest
C+ =
k Borrowed amount - interest
18
.Answer (D) is correct. Companies hold cash to facilitate routine transactions, to compensate banks for providing loans
and services, to guard against unforeseen fluctuations in cash flows, and to take advantage of opportunities for bargain
purchases. However, firms are more likely to hold trading securities or rely on easy access to credit to take advantage of
bargains or to guard against unforeseen fluctuations in cash flows.
Answer (A) is incorrect because a transactions balance is associated with routine payments and collections. Answer (B)
is incorrect because a compensating balance is the minimum amount on deposit at a bank to compensate the bank for
providing loans and other services. Answer (C) is incorrect because a precautionary balance consists of reserves for
unforeseen fluctuations in cash flows.
2(F)(T) Interest
C+ =
k Borrowed amount - interest
19
.Answer (D) is correct. A company will hold cash and marketable securities to facilitate business transactions because
cash is a primary medium of exchange. Cash and near-cash items are also held to meet future needs, to satisfy
compensating balance requirements imposed by lenders, and to provide a precautionary balance for security purposes.
Cash is usually not held in an attempt to earn maximum returns on investment because cash and marketable securities
are not usually the highest-paying investments.
Answer (A) is incorrect because cash is held to satisfy compensating balance requirements. Answer (B) is incorrect
because cash is held to facilitate transactions. Answer (C) is incorrect because cash is held to meet future needs.

2(F)(T) Interest
C+ =
k Borrowed amount - interest
20
.Net float = Disbursements float - Collections float; therefore the larger the disbursements float and the lower the
collections float the better the cash management system. A lockbox is used to speed cash collections. If a firm’s
outflows come due early in the month rather than uniformly this will necessitate a large line of credit.

2(F)(T) Interest
C+ =
k Borrowed amount - interest
21
.Answer (D) is correct. A lock-box system accelerates the inflow of funds. A company maintains mail boxes, often in
numerous locations around the country, to which customers send payments. A bank checks these mailboxes several
times a day, and funds received are immediately deposited to the company's account without first being processed by
the company's accounting system, thereby hastening availability of the funds.
Answer (A) is incorrect because a lock-box system is not related to compensating balances; a compensating balance
may be required by a covenant in a loan agreement that requires a company to maintain a specified balance during the
term of the loan. Answer (B) is incorrect because a lock-box system is a process by which payments are sent to a
bank's mailbox, which is checked during normal post office hours. Answer (C) is incorrect because the use of a lock-box
system entails sending checks through the mail to a post office box. Thus, it does not reduce the risk of losing checks in
the mail.

2(F)(T) Interest
C+ =
k Borrowed amount - interest
22
.REQUIRED: The working capital technique that delays the outflow of cash.
DISCUSSION: A draft is a three-party instrument in which one person (the drawer) orders a second person (the
drawee) to pay money to a third person (the payee). A check is the most common form of draft. It is an instrument
payable on demand in which the drawee is a bank. Consequently, a draft can be used to delay the outflow of cash. A
draft can be dated on the due date of an invoice and will not be processed by the drawee until that date, thereby
eliminating the necessity of writing a check earlier than the due date or using an EFT. Thus, the outflow is delayed until
the check clears the drawee bank.
Answer (A) is incorrect because factoring is the sale of receivables and therefore concerns cash inflows, not outflows.
Answer (B) is incorrect because a lockbox system is a means of accelerating cash inflows. Answer (D) is incorrect
because an electronic funds transfer results in an immediate deduction from the payor’s bank account, thereby
eliminating float.

2(F)(T) Interest
C+ =
k Borrowed amount - interest
23
.Answer (B) is correct. Payment by draft, a three-party instrument in which the drawer orders the drawee to pay money
to the payee, is a means of slowing cash outflows. A check is the most common type of draft. Check float arises from
the delay between an expenditure and the clearing of the check through the banking system.
Answer (A) is incorrect because concentration banking, a lockbox system, and the use of a local post office box are
techniques used to accelerate cash receipts. Answer (C) is incorrect because EDI is the communication of electronic
documents directly from a computer in one entity to a computer in another entity. Thus, EDI expedites cash payments.
The payee receives the money almost instantaneously. Answer (D) is incorrect because concentration banking, a
lockbox system, and the use of a local post office box are techniques used to accelerate cash receipts.

2(F)(T) Interest
C+ =
k Borrowed amount - interest
24
.REQUIRED: The definition of an automated clearing house (ACH) electronic transfer.
DISCUSSION: (A) An ACH electronic funds transfer (EFT) is an electronic payment to a company’s account of a
concentration bank. A concentration bank is a large bank to which a company transfers funds from local depository
banks. These local banks operate the company’s lockboxes and thus serve as collection points. The transfer of funds
to the concentration bank allows the company to take advantage of economies of scale in cash management. The use
of an ACH facilitates concentration banking. ACHs are electronic networks operated by the Federal Reserve (except for
the New York regional ACH association) that guarantee 1-day clearing.
Answer (B) is incorrect because a check is not involved in an EFT. Answer (C) is incorrect because an ACH transfer
involves the actual transfer of funds electronically; it is not just a computer generated document. Answer (D) is incorrect
because an EFT is not a check-like instrument.

2(F)(T) Interest
C+ =
k Borrowed amount - interest
25
.Answer (C) is correct. Various methods of accelerating cash collections include decentralized collection outposts
(normally one in each Federal Reserve District), electronic funds transfers, centralized banking for all company branches
to avoid having to maintain minimum balances in several locations, and lockbox systems. A compensating balance is a
minimum average or absolute amount that must be maintained in a bank account. Hence, it is not a means of
accelerating cash collections. This requirement means that less cash is available to the depositor.
Answer (A) is incorrect because it is a common method of accelerating cash collections. Answer (B) is incorrect
because it is a common method of accelerating cash collections. Answer (D) is incorrect because it is a common
method of accelerating cash collections.

2(F)(T) Interest
C+ =
k Borrowed amount - interest
26
.Answer (D) is correct. Compensating balances are either (1) an absolute minimum balance or (2) a minimum average
balance that bank customers must keep at the bank. These are generally required by the bank to compensate for the
cost of services rendered. Maintaining compensating balances will not accelerate a company's cash inflows because
less cash will be available even though the amount of cash coming in remains unchanged.
Answer (A) is incorrect because multiple collection centers throughout the country will reduce the time required to
receive cash in the mail. For example, California customers of a New York firm would make payment to a West Coast
center. Thus, the company would receive the cash two or three days sooner. Answer (B) is incorrect because direct
deposit by customers into a lock-box also speeds cash into company accounts. Answer (C) is incorrect because special
handling of large checks is a cost-effective way to deposit large amounts.

2(F)(T) Interest
C+ =
k Borrowed amount - interest
27
.Answer (A) is correct. Cash conversion is the process of converting cash to products and back to cash. The length of
the cycle is from the cash outflow to the cash inflow. Thus, the cash conversion cycle, in the case of production, begins
with the cash purchase of raw materials and ends with the collection of accounts receivable.
Answer (B) is incorrect because the cash inflows have not been received. Answer (C) is incorrect because the cash
inflows have not been received. Answer (D) is incorrect because the cash inflows have not been received.
2(F)(T) Interest
C+ =
k Borrowed amount - interest
28
.Statement a is false. If inventory increases, and sales do not, more cash is being “tied up” in inventory so the cash
conversion cycle is increased, not reduced. Statement b is true. If the company reduces its DSO, it is collecting its
accounts receivables more efficiently, so it reduces the cash conversion cycle. Statement c is false. If the company pays
its bills sooner, it uses its cash to pay off accounts payable, which increase its cash conversion cycle.

2(F)(T) Interest
C+ =
k Borrowed amount - interest
29
.Statements a and b are correct; therefore, statement d is the appropriate choice. Delaying payments to suppliers
increases the length of the cash conversion cycle.

2(F)(T) Interest
C+ =
k Borrowed amount - interest
30
.Answer (D) is correct. Cash and short-term investments are crucial to a firm's continuing success. Sufficient liquidity
must be available to meet payments as they come due. At the same time, liquid assets are subject to significant control
risk. Therefore, liquidity and safety are the primary concerns of the treasurer when dealing with highly liquid assets.
Cash and short-term investments are held because of their ability to facilitate routine operations of the company. These
assets are not held for purposes of achieving investment returns.
Answer (A) is incorrect because most companies are not in business to earn high returns on liquid assets (i.e., they are
held to facilitate operations). Answer (B) is incorrect because the holding of cash and cash-like assets is not a major
factor in controlling taxes. Answer (C) is incorrect because investments in Treasury bonds do not have sufficient liquidity
to serve as short-term assets.

2(F)(T) Interest
C+ =
k Borrowed amount - interest
31
.Answer (C) is correct. The trade-off between risk and return must be considered because liquid assets are usually less
profitable than less-liquid alternatives. However, a greater liquidity means less risk of being unable to meet obligations
when they are due.
Answer (A) is incorrect because financial leverage concerns the extent to which debt financing is used. Answer (B) is
incorrect because operating leverage concerns the proportion of fixed operating costs. Answer (D) is incorrect because
the costs of issuing securities relate to capital structure finance.

2(F)(T) Interest
C+ =
k Borrowed amount - interest
32
.REQUIRED: The true statement about cash management models.
DISCUSSION: (D) Because cash and inventory are both non-earning assets, in principle they may be treated similarly.
The alternative to holding cash, however, is to hold marketable securities that do earn interest or dividends. Thus, a
cash management model determine how much of a firm’s liquidity should be held as cash and how much in the form of
marketable securities.
Answer (A) is incorrect because credit and collection policies concern receivables and are not influenced by an EOQ
model for inventory management. Answer (B) is incorrect because the level of marketable securities is in part
determined by cash needs, so (D) is a better answer. Answer (C) is incorrect because the relationship between current
assets and current liabilities concerns many factors other than cash management.

2(F)(T) Interest
C+ =
k Borrowed amount - interest
33
.Answer (A) is correct. The optimal amount of cash to be raised by selling securities is calculated by a formula similar to
that used to determine the economic order quantity for inventory.
2(F)(T)
C+ =
k
If: C+ = Cash to be raised
T = Total cash needed for the period
F = Cost of making a securities trade
k = Opportunity cost of holding cash
The optimal amount of cash to be raised by selling securities is inversely related to the rate of return forgone
(opportunity cost) and directly related to the cost of the transaction.
Answer (B) is incorrect because a high (low) opportunity cost results in a lower (higher) optimal cash balance, whereas
high (low) transaction costs result in a higher (lower) optimal cash balance. Answer (C) is incorrect because a high (low)
opportunity cost results in a lower (higher) optimal cash balance, whereas high (low) transaction costs result in a higher
(lower) optimal cash balance. Answer (D) is incorrect because a high (low) opportunity cost results in a lower (higher)
optimal cash balance, whereas high (low) transaction costs result in a higher (lower) optimal cash balance.

Interest
Borrowed amount - interest
2(F)(T)
C+ =
34 k .REQUIRED: The assumption not made in the EOQ model.
DISCUSSION: (D) The EOQ formula is a deterministic model that requires a known demand for inventory, or in this
case, the amount of cash needed. Thus, the cash flow requirements cannot be random. The model also assumes a
given carrying (interest) cost and a flat transaction cost for converting marketable securities to cash, regardless of the
amount withdrawn.
Answer (A) is incorrect because the EOQ model assumes that the cost of a transaction is independent of the dollar
amount of the transaction and interest rates are constant over the short-run. Answer (B) is incorrect because the EOQ
model assumes an opportunity cost is associated with holding cash, beginning with the first dollar. Answer (C) is
incorrect because the EOQ model assumes that the total demand for cash is known with certainty.

Interest
Borrowed amount - interest
2(F)(T)
C+ =
35 k .Answer (A) is correct. Risk and yield are fundamental concerns, and taxation is a pervasive issue
that affects yield. Default risk is the risk that a borrower will be unable to make interest payments or principal
repayments on debt (e.g., bonds purchased from a financially troubled company have high default risk). Taxability refers
to the tax treatment of any income or loss from a security. Relative yields are the rates of return of each security in
comparison with other potential investments.
Answer (B) is incorrect because these matters are a financial manager's overall responsibility, not securities selection
criteria. Answer (C) is incorrect because these cycles are important in working capital management. The operating and
payment cycles are components of the cash conversion cycle. This cycle lasts from the time cash is paid for resources
to the time cash is received in payment for goods sold. Answer (D) is incorrect because these are factors in the
determination of cash flow. The inventory conversion cycle is the time between acquisition of resources and sale of
finished goods. The receivables conversion cycle lasts from the date of sale of finished goods to the date of cash
collection. A payable deferral period is the lapse of time between purchase of resources and the date they are paid for.
Thus, the cash conversion cycle does not begin until the end of the payable deferral period.

Interest
Borrowed amount - interest
2(F)(T)
C+ =
36 k .Answer (A) is correct. A Treasury bill is a short-term U.S. government obligation that is sold at a
discount from its face value. A Treasury bill is highly liquid and nearly risk-free, and it is often held as a substitute for
cash.
Answer (B) is incorrect because common stock lacks the liquidity necessary to be a cash substitute. Common stock can
also be a risky investment. Answer (C) is incorrect because gold lacks the liquidity necessary to be a cash substitute.
Gold can also be a risky investment. Answer (D) is incorrect because Aaa corporate bonds lack the liquidity necessary
to be a cash substitute. Aaa corporate bonds can also be risky investments.

Interest
Borrowed amount - interest
2(F)(T)
C+ =
37 k .Answer (B) is correct. The marketable securities with the lowest default risk are those issued by the
federal government because they are backed by the full faith and credit of the U.S. Agency securities are issued by
agencies and corporations created by the federal government, such as the Federal Housing Administration. They are
backed by a secondary promise from the government.
Answer (A) is incorrect because securities issued by a federal agency are first backed by that agency and secondarily
by the U.S. government. Answer (C) is incorrect because repurchase agreements could become worthless if the
organization agreeing to make the repurchase goes bankrupt. Answer (D) is incorrect because commercial paper is
unsecured.

Interest
Borrowed amount - interest
2(F)(T)
C+ =
38 k .Only Treasury issues are insured by the U. S. government; the shorter-term the instrument, the safer
the instrument.

Interest
Borrowed amount - interest
2(F)(T)
C+ =
39 k .Answer (C) is correct. Funds are borrowed or lent for short periods (less than one year) in money
markets. Examples of instruments traded in money markets are U.S. Treasury bills, bankers' acceptances, commercial
paper, negotiable certificates of deposit, money market mutual funds, Eurodollar market time deposits, and consumer
credit loans. Capital markets trade shares and long-term debt.
Answer (A) is incorrect because mortgages are long-term capital market securities. Answer (B) is incorrect because
preferred shares are long-term capital market securities. Answer (D) is incorrect because U.S. Treasury bonds are long-
term capital market securities.

Interest
Borrowed amount - interest
2(F)(T)
C+ =
40 k .Answer (D) is correct. A certificate of deposit (CD) is a form of savings deposit that cannot be
withdrawn before maturity without incurring a high penalty. A negotiable CD can be traded. CDs usually have a fairly
high rate of return compared with other savings instruments because they are for fixed, usually long-term periods.
However, their yield is less than that of commercial paper and bankers' acceptances because they are less risky.
Answer (A) is incorrect because negotiable CDs do have a secondary market (i.e., they are negotiable). Answer (B) is
incorrect because negotiable CDs are regulated. Answer (C) is incorrect because negotiable CDs are typically issued in
a denomination of $100,000.

Interest
Borrowed amount - interest
2(F)(T)
C+ =
41 k .Answer (C) is correct. Of the choices given, the commercial paper of a top-rated (most creditworthy)
company has the least risk. Commercial paper is preferable to stock or stock options because the latter represent only a
residual equity in a corporation. Commercial paper is debt and thus has priority over stockholders' claims. Also,
commercial paper is a very short-term investment. The maximum maturity allowed without SEC registration is 270 days.
However, it can be sold only to sophisticated investors without registration.
Answer (A) is incorrect because municipal bonds are rarely considered marketable securities in that they constitute long-
term debt. Answer (B) is incorrect because common stock does not have as high a priority in company assets as
commercial paper or other debt. Answer (D) is incorrect because common stock does not have as high a priority in
company assets as commercial paper or other debt.

Interest
Borrowed amount - interest
2(F)(T)
C+ =
42 k .Answer (C) is correct. A small firm with surplus cash should invest for the highest return and lowest
risk. The ability to convert the investment into cash without a loss of principal is also important. Money market mutual
funds invest in money market certificates such as treasury bills, negotiable CDs, and commercial paper. Because of
diversification, these mutual funds are superior to any single instrument.
Answer (A) is incorrect because a small firm may not have enough surplus cash to invest in commercial paper, which
usually consists of secured or unsecured promissory notes of large corporations. Answer (B) is incorrect because the
transactions cost of bankers' acceptances is high. A banker's acceptance is a unique credit instrument used to finance
both domestic and international "self-liquidating" transactions. It is usually initiated by a bank's irrevocable letter of credit
on behalf of the bank's customer, on which the company doing business with the bank's customer draws a time draft.
The company discounts the time draft with the company's local bank and receives immediate payment. The local bank
forwards the time draft to the bank customer for payment. Answer (D) is incorrect because an increase in interest rates
could cause a substantial loss in principal.

Interest
Borrowed amount - interest
2(F)(T)
C+ =
43 k .Answer (B) is correct. Eurobonds are, by definition, always sold in some country other than the one
in whose currency the bond issue is denominated. Their advantage is that they are customarily less stringently regulated
than most other bonds. Hence, transaction costs are lower.
Answer (C) is incorrect because foreign bonds are denominated in the currency of the country in which they are sold.
(A) is incorrect because Eurobonds are not always denominated in Eurodollars, which are U.S. dollars deposited outside
the U.S. Answer (D) is incorrect because Eurobonds are usually issued not as registered bonds but as bearer bonds, so
names and nationalities of the investors are not recorded.

Interest
Borrowed amount - interest
2(F)(T)
C+ =
44 k .Answer (C) is correct. Eurodollars are U.S. dollars on deposit in a foreign bank. These deposits are
created when a check is drawn on a dollar deposit in a U.S. bank and then deposited in a bank outside the U.S. This
amount is then available for lending by the foreign bank to its customers. However, the depositors still hold claims
denominated in dollars. Because Eurodollars are outside the direct control of the U.S. monetary authorities, U.S.
banking regulations with respect to reserves, insurance, interest ceilings, etc., do not apply. The absence of these costs
means that Eurodollar deposit rates tend to be higher, not lower, than domestic U.S. rates on equivalent instruments.
Answer (A) is incorrect because these deposits are outside the control of U.S. authorities because they are in banks
outside the U.S. Answer (B) is incorrect because these deposits are outside the control of U.S. authorities because they
are in banks outside the U.S. Answer (D) is incorrect because interest rates on these deposits, and loans made thereon,
are tied to the LIBOR, which is the rate paid on deposits of other large banks by the largest London banks with the
highest credit standing.

Interest
Borrowed amount - interest
2(F)(T)
C+ =
45 k .Answer (D) is correct. Eurodollars are U.S. dollars deposited in banks outside the U.S. Because it is
outside the direct control of the U.S. monetary authorities, the Eurodollar market has lower costs. For example, U.S.
reserve requirements and FDIC premium payments do not apply in this market. A lower cost market can offer depositors
higher interest rates.
Answer (A) is incorrect because Eurodollar borrowers tend to pay lower, not higher, rates. Borrowers and depositors can
both receive more favorable rates because, with its lower costs, the Eurodollar market can offer smaller spreads
between borrowing and lending rates. Answer (B) is incorrect because U.S. dollars are on deposit in both cases.
Answer (C) is incorrect because Eurodollar deposits tend to be for larger, not smaller, amounts. Furthermore, smaller
deposits tend to earn lower, not higher, rates than larger deposits.

Interest
Borrowed amount - interest
2(F)(T)
C+ =
46 k .Answer (D) is correct. Marketable securities are near-cash items used primarily for short-term
investment. Examples include U.S. Treasury bills, Eurodollars, commercial paper, money-market mutual funds with
portfolios of short-term securities, bankers' acceptances, floating-rate preferred stock, and negotiable CDs of U.S.
banks. A convertible bond is not a short-term investment because its maturity date is usually more than one year in the
future and its price can be influenced substantially by changes in interest rates or by changes in the investee's stock
price.
Answer (A) is incorrect because U.S. Treasury bills are short-term marketable securities. Answer (B) is incorrect
because Eurodollars are short-term marketable securities. Answer (C) is incorrect because commercial paper is a short-
term marketable security.

Interest
Borrowed amount - interest
2(F)(T)
C+ =
47 k .REQUIRED: The definition of “short-selling”
DISCUSSION: (B) Short-selling is accomplished by borrowing securities from a broker and selling those securities. At
a later time, the loan is repaid by buying securities on the open market and returning them to th broker. The seller
speculates that the stock’s market price will decline.
Answer (A) is incorrect because margin trading involves buying securities bu borrowing money from a broker. Answer
(C) is incorrect because the investor does not own the shares sold in a short sale. Answer (D) is incorrect because the
short seller is betting that the stock will decrease in price.

Interest
Borrowed amount - interest
2(F)(T)
C+ =
48 k .Answer (C) is correct. The day's sales outstanding (days of receivables) may be stated as the
accounts receivable balance divided by average credit sales per day or as days in the year divided by the receivables
turnover. It is the average time required to convert the enterprise's receivables into cash. Thus, it is also called the
receivables collection period.
Answer (A) is incorrect because the inventory conversion period (days of inventory) is the average time required to
convert materials into finished goods and then to sell them. This process typically occurs before the receivables
collection period, and the amount of time in one period does not necessarily bear any relationship to the other. Answer
(B) is incorrect because the cash conversion cycle equals the inventory conversion period, plus the receivables
collection period, minus the payables deferral period (average time between resource purchases and payment of cash
for them). It estimates the time between when the enterprise makes payments and when it receives cash inflows.
Answer (D) is incorrect because the inventory divided by the sales per day is the inventory conversion period (days of
inventory).

Interest
Borrowed amount - interest
2(F)(T)
C+ =
49 k .REQUIRED: The meaning of a firm’s average collection period.
DISCUSSION: (A) The average collection period measures the number of days between the date of sale and the date
of collection. It should be related to a firm’s credit terms. For example, a firm that allows terms of 2/15, net 30 should
have an average collection period of somewhere between 15 and 30 days.
Answer (B) is incorrect because it describes the concept of float. Answer (C) is incorrect because the average collection
period includes the total time before a payment is received. Including the periods both before and after the end of the
normal credit period. Answer (D) is incorrect because it describes the normal credit period.

Interest
Borrowed amount - interest
2(F)(T)
C+ =
50 k .REQUIRED: The item measured by an aging of accounts receivable.
DISCUSSION: (D) The purpose of an aging of receivables is to classify receivables by due date. Those that are
current (not past due) are listed in one column, those less than 30 days past due in another column, etc. The amount in
each category can then be multiplied by an estimated bad debt percentage that is based on a company’s credit
experience and other factors. The theory is that the oldest receivables are the least likely to be collectible. Aging the
receivables and estimating the uncollectible amounts is one method of arriving at the appropriate balance sheet
valuation of the accounts receivable account.
Answer (A) is incorrect because an aging schedule is used for receivables, not liabilities. Answer (B) is incorrect
because an aging schedule concerns specific accounts, not averages. Answer (C) is incorrect because an aging
schedule focuses on uncollectible receivables.

Interest
Borrowed amount - interest
2(F)(T)
C+ =
51 k .Answer (B) is correct. A quantity discount is an attempt to increase sales by reducing the unit price
on bulk purchases. It concerns only the price term of an agreement, not the credit term, and thus is unrelated to credit
and collection policy.
Answer (A) is incorrect because the quality of accounts is important to credit policy since it is inversely related to both
sales and bad debts. Answer (C) is incorrect because offering a cash discount improves cash flow and reduces
receivables and the cost of extending credit. Answer (D) is incorrect because the level of collection expenditures must
be considered when implementing a collection policy. The marginal cost of a credit and collection policy should not
exceed its revenue.

Interest
Borrowed amount - interest
2(F)(T)
C+ =
52 k .Answer (D) is correct. Increasing the quality of the accounts rejected means that fewer sales will be
made. The company is therefore not trying to maximize its sales or increase its bad debt losses. The objective is to
reduce bad debt losses and thereby maximize profits.
Answer (A) is incorrect because tightening credit will reduce sales and bad debt losses. Answer (B) is incorrect because
tightening credit will reduce sales and bad debt losses. Answer (C) is incorrect because, most likely, higher quality
accounts will mean a shorter average collection period.

Interest
Borrowed amount - interest
2(F)(T)
C+ =
53 k .REQUIRED: The calculation of the average gross receivable balance.
DISCUSSION: A firm’s average gross receivables balance can be calculated by multiplying average daily sales by the
average collection period (days’ sales outstanding). Alternatively, annual credit sales can be divided by the accounts
receivable turnover (net credit sales ÷ average accounts receivable) to obtain the average balance in receivables.
Answers (A) and (B) are incorrect because Alternative 1 cannot be correct. Neither of the multiplicands is a dollar figure,
sot the product could not be the dollar balance of receivables. Answer (D) is incorrect because Alternative III cannot be
correct. It contains average gross receivables, the amount being calculated.

Interest
Borrowed amount - interest
2(F)(T)
C+ =
54 k .Answer (C) is correct. A high receivables turnover is indicative of short credit terms and possibly the
use of a lockbox system to speed up cash flows.
Answer (A) is incorrect because a lenient credit policy would lead to a low turnover. Answer (B) is incorrect because
small discounts although helpful, would be unlikely to lead to an unusually high turnover. Small discounts would not be
as helpful as a lockbox system and short credit terms. Answer (D) is incorrect because a high turnover would be
indicative of an efficient credit and collection department.

Interest
Borrowed amount - interest
2(F)(T)
C+ =
55 k .Answer (B) is correct. A decline in the receivable turnover indicates that either sales are declining
relative to receivables, or receivables are increasing relative to sales. Thus, the firm is collecting credit sales more slowly
than before.
Answer (A) is incorrect because sales are being collected more slowly when the turnover declines. Answer (C) is
incorrect because an increase in sales will increase the turnover ratio. Answer (D) is incorrect because inventories have
no impact on the receivable turnover ratio.

Interest
Borrowed amount - interest
2(F)(T)
C+ =
56 k . Answer (D) is correct. The accounts receivable turnover ratio equals net credit sales divided by
average receivables. Hence, it will decrease if a company lengthens the credit period or the discount period because the
denominator will increase as receivables are held for longer times.
Answer (A) is incorrect because write-offs do not reduce net receivables (gross receivables - the allowance) and will not
affect the receivables balance and therefore the turnover ratio if an allowance system is used. Answer (B) is incorrect
because a decline in sales near the end of the period signifies fewer credit sales and receivables, and the effect of
reducing the numerator and denominator by equal amounts is to increase the ratio if the fraction is greater than 1.0.
Answer (C) is incorrect because an increase in cash sales with no diminution of credit sales will not affect receivables.

Interest
Borrowed amount - interest
2(F)(T)
C+ =
57 k .REQUIRED: The reason for offering credit terms of 2.10, net 30.
DISCUSSION: (D) Because these terms involve an annual interest cost of over 36%, a company would not offer them
unless it desperately needed cash. Also, credit terms are typically somewhat standardized within an industry. Thus if
most companies in the industry offer similar terms, a firm will likely be forced to match the competition or lose market
share.
Answer (A) is incorrect because, if the company does not need cash, it would not offer cash discounts, regardless of its
cost of capital, unless required to match competition. Answer (B) is incorrect because the ability to borrow at a lower
rate is a reason for not offering cash discounts. Answer (C) is incorrect because the relationship between the cost of
capital and the prime rate may not be relevant if the firm cannot borrow at the prime rate.

Interest
Borrowed amount - interest
2(F)(T)
C+ =
58 k .REQUIRED: The true statement about extending credit for a period longer than the purchase’s
operating cycle.
DISCUSSION: (B) The normal operating cycle is defined as the period from the acquisition of inventory to the collection
of the account receivable. If trade credit is for a period longer than the normal operating cycle, the seller must therefore
be financing more than just the purchase of inventory.
Answer (A) is incorrect because a seller that extends long-term credit will have a higher level of receivables than a firm
with a shorter credit period. Answer (C) is incorrect because the seller is not guaranteed that a purchaser will resell the
merchandise. Answer (D) is incorrect because offering a discount may accelerate payment.

Interest
Borrowed amount - interest
2(F)(T)
C+ =
59 k .Answer (C) is correct. If the cause of increased sales is an increase in the cash discount, it can be
inferred that the additional customers would pay during the discount period. Thus, cash would be collected more quickly
than previously and the cash conversion cycle would be shortened.
Answer (A) is incorrect because the operating cycle would decrease since the average time from cash disbursement to
cash realization would be shorter. Answer (B) is incorrect because the average collection period would decrease since
the average time from cash disbursement to cash realization would be shorter. Answer (D) is incorrect because more
customers will take discounts.

Interest
Borrowed amount - interest
2(F)(T)
C+ =
60 k .REQUIRED: The true statement about a change in credit policy that has resulted in greater sales
and a reduction in accounts receivable.
DISCUSSION: (A) An increase in discounts taken accompanied by declines in receivables balances and doubtful
accounts all indicate that collections on the increased sales have been accelerated. Accordingly, the average collection
period is a ratio calculated by dividing the number of days in a year (365) by the receivable turnover. Thus, the higher
the turnover, the shorter the average collection period. The turnover increases when either sales (the numerator)
increase or receivables (the denominator) decrease. Accomplishing both higher sales and a lower receivables
increases the turnover and results in a shorter collection period.
Answer (B) is incorrect because a decrease in the percentage discount offered provides no incentive for early payment.
Answer (C) is incorrect because accounts receivable turnover (sales ÷ average receivables) has increased. Answer (D)
is incorrect because no information is given relative to working capital elements other than receivables. Both
receivables and cash are elements of working capital, so an acceleration of customer payments will have no effect on
working capital.

Interest
Borrowed amount - interest
2(F)(T)
C+ =
61 k .REQUIRED: The true statement about a change in credit policy that has resulted in greater sales
and a reduction in accounts receivable.
DISCUSSION: (B) An increase in discounts taken accompanied by declines in receivables balances and doubtful
accounts all indicate that collections on the increased sales have been accelerated. Accordingly, the average collection
period must have declined. The average collection period is a ratio calculated by dividing the number of days in a year
(365) by the receivable turnover. Thus, the higher the turnover, the shorter the average collection period. The turnover
increases when either sales (the numerator) increase, or receivables (the denominator) decrease. Accomplishing both
higher sales and a lower receivables increases the turnover and results in a shorter collection period.
Answers (A) and (C) are incorrect because no statement can be made with respect to profits without knowing costs.
Answer (D) is incorrect because the discount may have been increased, which has led to quicker payments. Answer (E)
is incorrect because the bad debt loss percentage has apparently declined. Sales are up; doubtful accounts are fewer.

Interest
Borrowed amount - interest
2(F)(T)
C+ =
62 k .REQUIRED: The factor not considered in an analysis of propose credit policies.
DISCUSSION: (B) All factors should be considered that differ between the two policies. Factors that do not differ, such
as the current bad debt experience, are not relevant. Ryan must estimate the expected bad debt losses under each
new policy.
Answer (A) is incorrect because the cost of funds is an obvious element in the analysis of any investment. Answer (C) is
incorrect because the impact on the current customer base of extending terms to only certain customers is relevant.
The current customers may demand the same terms. Answer (D) is incorrect because existing loan agreements may
require Ryan to maintain certain ratios at stated levels. Thus, Ryan’s ability to increase receivables and possible bad
debt losses may be limited.

Interest
Borrowed amount - interest
2(F)(T)
C+ =
63 k .Answer (A) is correct. Relaxing the credit policy for customers will lead to increased sales because
more people will be eligible for more credit. As sales increase, purchase orders will increase to accommodate the higher
sales levels.
Answer (B) is incorrect because inventory should be increased to accommodate higher sales levels. Answer (C) is
incorrect because safety stock is based on expected sales, which are expected to rise. Answer (D) is incorrect because
a just-in-time system is not used when a company orders inventory once a quarter.

Interest
Borrowed amount - interest
2(F)(T)
C+ =
64 k .Answer (A) is correct. A factor purchases a company's accounts receivable and assumes the risk of
collection. The seller receives money immediately to reinvest in new inventories. The financing cost is usually high:
about 2 points or more above prime, plus a fee for collection. Factoring has been traditional in the textile industry for
years, and recently companies in many industries have found it an efficient means of operation. A company that uses a
factor can eliminate its credit department, accounts receivable staff, and bad debts. These reductions in costs can more
than offset the fee charged by the factor, which can often operate more efficiently than its clients because of the
specialized nature of its service.
Answer (B) is incorrect because factoring is a source of short-term funds through sale of receivables. Answer (C) is
incorrect because factoring is a source of short-term funds through sale of receivables. Answer (D) is incorrect because
factoring is a source of short-term funds through sale of receivables.

Interest
Borrowed amount - interest
2(F)(T)
C+ =
65 k .Answer (B) is correct. Pledging accounts receivable is an arrangement in which receivables are
used as security for a loan. But if the receivables are not paid, the secured party still may proceed against the
borrowers. Lenders in these circumstances choose the collateral with care, retain the right of recourse against the
borrower in case of default, and usually lend some amount less than the face value of the receivables.
Answer (A) is incorrect because factoring is the outright sale of receivables for cash at a discount. Answer (C) is
incorrect because receivables may be sold with or without recourse. Answer (D) is incorrect because hedging is the
process of protecting oneself against loss because of future price changes. For example, a party that has agreed to
deliver a commodity at a certain date may enter into a hedging contract to buy an equal quantity on the same date. Any
loss on one transaction should then be offset by the gain on the other.

Interest
Borrowed amount - interest
2(F)(T)
C+ =
66 k .Answer (A) is correct. An increase in the proportion of short-term financing will not affect a
company's degree of leverage, but risk is increased because of the need for frequent refinancing. Because the debtor
company will be forced to meet principal and interest payments quickly, perhaps before expected funds from a new
project, the danger of default is increased. Also, future interest rates are difficult to predict.
Answer (B) is incorrect because leverage is the use of borrowed funds to earn returns for stockholders. It is irrelevant
whether the borrowed funds are long- or short-term. Answer (C) is incorrect because the length of a loan does not affect
the amount of liquid assets. Both long- and short-term loans result in liquid assets. Answer (D) is incorrect because an
increase in current liabilities decreases the current ratio.

Interest
Borrowed amount - interest
2(F)(T)
C+ =
67 k .Under normal conditions the yield curve is upward sloping, thus, short-term interest rates are lower
than long-term interest rates. Consequently, a firm financing with short-term debt will pay less interest than a firm
financing with long-term debt--increasing its ROE. However, a firm increases its risk by financing with short-term debt
because such debt must be “rolled over” frequently, and the firm is exposed to the volatility of short-term rates. The
other statements are false.

Interest
Borrowed amount - interest
2(F)(T)
C+ =
68 k .REQUIRED: The item that provides a spontaneous source of financing.
DISCUSSION: (A) Trade credit is a spontaneous source of financing because it arises automatically as part of a
purchase transaction. Because of its ease in use, trade credit is the largest source of short-term financing for may firms
both large and small.
Answers (B) and (D) are incorrect because mortgage bonds and debentures do not arise automatically as a result of a
purchase transaction. Answer (C) is incorrect because the use of receivables as a financing source requires an
extensive factoring arrangement and often involves the creditor’s evaluation of the credit ratings of the borrower’s
customers.

Interest
Borrowed amount - interest
2(F)(T)
C+ =
69 k .Answer (D) is correct. Trade credit is a spontaneous source of financing because it arises
automatically as part of the purchase transaction.
Answer (A) is incorrect because it occurs as a result of transactions apart from purchase transactions. In other words,
such credit is arranged separately from the transactions to acquire the assets being financed. Answer (B) is incorrect
because it occurs as a result of transactions apart from purchase transactions. In other words, such credit is arranged
separately from the transactions to acquire the assets being financed. Answer (C) is incorrect because prepaid interest
is not a source of financing.

Interest
Borrowed amount - interest
2(F)(T)
C+ =
70 k .Answer (C) is correct. Trade credit is a spontaneous source of financing because it arises
automatically as part of a purchase transaction. Because of its ease in use, trade credit is the largest source of short-
term financing for many firms both large and small.
Answer (A) is incorrect because installment loans are usually a longer-term source of financing and are more difficult to
acquire than trade credit. Answer (B) is incorrect because commercial paper is normally used only by large companies
with high credit ratings. Answer (D) is incorrect because mortgage bonds are a long-term source of financing.

Interest
Borrowed amount - interest
2(F)(T)
C+ =
71 k .REQUIRED: The true statement about trade credit.
DISCUSSION: (C) Trade credit is a spontaneous source of financing because it arises automatically as part of a
purchase transaction. The terms of payment are set by the supplier, but trade credit usually requires payment within a
short period of time. Trade credit is an important source of credit for all businesses but especially for buyers, such as
small businesses but especially for buyers, such as small businesses, that might not have access to other credit
markets. Like all forms of financing, trade credit is subject to the risk of buyer default.
Answer (A) is incorrect because trade credit is an important source of financing for small firms. Answer (B) is incorrect
because trade credit is ordinarily short-term source of financing. Answer (D) is incorrect because the cost of trade credit
depends on the credit terms and the price paid. A seller with generous payment terms may charge a higher price for its
merchandise.

Interest
Borrowed amount - interest
2(F)(T)
C+ =
72 k .Answer (A) is correct. The first term of the formula represents the periodic cost of the trade
discount, calculated as the cost per unit of trade credit (discount %) divided by the funds made available by not taking
the discount (100 - discount %). The second term represents the number of times per year this cost is incurred. The
multiple of these terms is the approximate annual percentage cost of not taking the trade discount. A precise formula
would incorporate the effects of compounding when calculating the annual cost.
Answer (B) is incorrect because the denominator of the first term should represent the funds made available by not
taking the discount (100 - discount %). Answer (C) is incorrect because the first term is the reciprocal of the correct term.
Answer (D) is incorrect because the second term is the reciprocal of the correct term.

Interest
Borrowed amount - interest
2(F)(T)
C+ =
73 k .Answer (D) is correct. If the discount period is longer, the days of extra credit obtained by forgoing
the discount are fewer. Assuming other factors are constant, the result is that the cost of trade credit, that is, the cost of
not taking the discount, is greater.
Answer (A) is incorrect because the lower the discount percentage, the lower the opportunity cost of forgoing the
discount and using the trade credit financing. Answer (B) is incorrect because percentage financing cost is unaffected by
the purchase price of the items. Answer (C) is incorrect because percentage financing cost is unaffected by the
purchase price of the items.

Interest
Borrowed amount - interest
2(F)(T)
C+ =
74 k .REQUIRED: The true statement about cash discounts.
DISCUSSION: (D) Payments should be made within the discount periods if the cost of not taking discounts exceed the
firm’s cost of capital. For example, failing to take a discount when terms are 2/10, net 30 means that the firm is paying
an effective annual interest rate exceeding 36%. Thus the cost of not taking the discount is usually higher than the cost
of a bank loan.
Answer (A) is incorrect because the cost of not taking a discount when terms are 2/10, net 30 exceeds 36% annually,
which is higher than the prime rate has ever been. Answer (B) is incorrect because the buyer is paying the amount of
discount not taken in exchange for the extra 45 days of credit. Answer (C) is incorrect because paying 2% for 20 days
of credit is more expensive than paying 2% for 50 days of the same amount of credit.

Interest
Borrowed amount - interest
2(F)(T)
C+ =
75 k .Answer (C) is correct. A small retail store would not have access to major capital markets. In fact,
the only options available, outside of owner financing, are bank loans and a line of credit from suppliers. It is this latter
alternative that is most often used because it permits the store to finance inventories for 30 to 60 days without incurring
interest cost. A line of credit is an arrangement between a bank and a borrower in which the bank commits itself to lend
up to a certain maximum amount to the borrower in a given period.
Answer (A) is incorrect because only large companies with excellent credit ratings have access to the commercial paper
market. Answer (B) is incorrect because a retail store must have instant access to its inventory to provide continuous
services to customers. Thus, a terminal warehouse receipt loan would not be suitable because the inventory would not
be in the immediate possession of the seller. Answer (D) is incorrect because a chattel mortgage is most often used for
financing movable equipment. It is not well-suited to financing inventory of a small retailer with high turnover because of
the difficulty of identification.

Interest
Borrowed amount - interest
2(F)(T)
C+ =
76 k .Statement b illustrates an aggressive financing policy, not a conservative one.

Interest
Borrowed amount - interest
2(F)(T)
C+ =
77 k .Answer (D) is correct. The first step is to determine the actual annual percentage interest rate for
each of the four options. Assuming a $100 invoice, the Fort Company discount represents interest of $1 on a loan of $99
for 20 days (30-day credit period - 10-day discount period). The annual interest rate is 18.1818% [(360/20) periods x
($1/$99)]. The Riley Company discount represents an interest charge of $2 on a loan of $98; i.e., by not paying on the
15th day, the company will have the use of $98 for 45 days (60-day credit period - 15-day discount period). The number
of periods in a year would be 8 (360/45). The interest would be 16.326% ($2/$98 x 8 periods). The Shad loan would be
for $97 at a cost of $3. The loan would be for 75 days (90 - 15). Given 4.8 interest periods in a year (360/75), the annual
interest rate would be 14.845% ($3/$97 x 4.8). The bank loan was quoted at 14.75% on a discount basis. On a $100
note, the borrower would only receive $85.25, giving an interest rate of 17.302% ($14.75/$85.25). Thus, not paying
Shad, Inc.'s invoices on time would be the lowest cost source of capital, at a cost of 14.845%.
Answer (A) is incorrect because the actual annual percentage rate based on forgoing Shad's discount is 14.845%. This
is lower than the rate on the bank loan (17.302% because it is a discount loan), or the cost of forgoing the discounts
allowed by Fort Co. (18.182%) and Riley Co. (16.326%). Answer (B) is incorrect because the actual annual percentage
rate based on forgoing Shad's discount is 14.845%. This is lower than the rate on the bank loan (17.302% because it is
a discount loan), or the cost of forgoing the discounts allowed by Fort Co. (18.182%) and Riley Co. (16.326%). Answer
(C) is incorrect because the actual annual percentage rate based on forgoing Shad's discount is 14.845%. This is lower
than the rate on the bank loan (17.302% because it is a discount loan), or the cost of forgoing the discounts allowed by
Fort Co. (18.182%) and Riley Co. (16.326%).

Interest
Borrowed amount - interest
2(F)(T)
C+ =
78 k .REQUIRED: The true statement about short-term interest rates.
DISCUSSION: (A) Historically, one facet of the term structure of interest rates (the relationship of yield and time to
maturity) is that short-term interest rates have ordinarily been lower than long-term rates. One reason is that less risk is
involved in the short run. Moreover, future expectations concerning interest rates affect the term structure. Most
economists believe that a long-term interest rate is an average of future expected short-term interest rates. For this
reason, the yield curve will slope upward if future rates are expected to rise, downward if interest rates are anticipated to
fall, and remain flat if investors think the rate is stable. Future inflation is incorporated into this relationship. Another
consideration is liquidity preference. Investors in an uncertain world will accept lower rates on short-term investments
because of their greater liquidity, whereas business debtors often prefer to pay higher rates on long-term debt to avoid
the hazards of short-term maturities.
Answer (B) is incorrect because short-term rates are usually lower than long-term rates. Answer (C) is incorrect
because short-term rates are more likely to be greater than long-term rates if current levels of inflation are high. Answer
(D) is incorrect because long-term rates may be viewed as short-term rates adjusted by a risk factor.

Interest
Borrowed amount - interest
2(F)(T)
C+ =
79 k .Answer (D) is correct. The prime interest rate is the rate charged by commercial banks to their best
(the largest and financially strongest) business customers. It is traditionally the lowest rate charged by banks. However,
in recent years, banks have been making loans at still lower rates in response to competition from the commercial paper
market.
Answer (A) is incorrect because the prime rate has nothing to do with a commitment fee on a bank loan. Answer (B) is
incorrect because the effective rate on most companies' bank loans will be much higher than the prime rate. Answer (C)
is incorrect because the prime rate is a bank loan rate, not the rate on commercial paper.

Interest
Borrowed amount - interest
2(F)(T)
C+ =
80 k .Answer (C) is correct. When a firm borrows money from the bank, it is often required to keep a
certain percentage of the funds in the bank at all times. These compensating balances effectively increase the rate of
interest on the money borrowed from the bank.
Answer (A) is incorrect because the floating interest rate is not always higher. It should float up or down with the prime
rate. Answer (B) is incorrect because a restriction on a new issuance does not raise the interest rate on money
previously borrowed. Answer (D) is incorrect because, if a firm chooses not to use its full line of credit and is not
charged for the unused portion, the rate of interest on the portion used does not increase.

Interest
Borrowed amount - interest
2(F)(T)
C+ =
81 k .Answer (B) is correct. Simple interest is charged on the amount actually paid to the borrower. If
interest is charged on a simple basis, the full $20,000 face value of the loan is made available to the borrower.
Answer (A) is incorrect because interest is charged on a discount basis when it is deducted from the face value
borrowed. Answer (C) is incorrect because interest charged on a discount basis results in a deduction from the face
value. The borrower does not receive the full face value of the loan. Answer (D) is incorrect because interest is charged
on an add-on basis when the face value of the loan initially equals the borrowed amount plus the nominal interest
charge.

Interest
Borrowed amount - interest
2(F)(T)
C+ =
82 k .Answer (B) is correct. A minimum checking account balance that a firm must maintain with a
commercial bank is a compensating balance. A bank may require a borrower to keep a certain percentage of the face
value of a loan in the firm's account. This requirement raises the real rate of interest to the borrower.
Answer (A) is incorrect because the cash balance necessary for a firm to conduct day-to-day business is a transactions
balance. Answer (C) is incorrect because a cash balance held in reserve for random, unforeseen fluctuations in cash
inflows and outflows is a precautionary balance. Answer (D) is incorrect because a cash balance that is held to enable
the firm to take advantage of any bargain purchases that might arise is a speculative balance.
Interest
Borrowed amount - interest
2(F)(T)
C+ =
83 k .Answer (A) is correct. Banks sometimes require a borrower to keep a certain percentage of the face
amount of a loan in a non-interest-bearing checking account. This requirement raises the effective rate of interest paid
by the borrower. This greater rate compensates a bank for services provided and results in greater profitability for the
financial institution.
Answer (B) is incorrect because, in financial accounting, a valuation allowance is used to reflect losses on marketable
securities. Answer (C) is incorrect because a safety stock of inventory is held to avoid inventory stockouts. Answer (D) is
incorrect because large depositors may receive favorable treatment, but compensating balances are funds maintained
by loan recipients for the benefit of the lender.

Interest
Borrowed amount - interest
2(F)(T)
C+ =
84 k .Answer (C) is correct. The discounted interest rate is based on the amount borrowed but is paid in
advance. It is calculated using the following formula:
Interest
Borrowed amount - interest
Answer (A) is incorrect because it is the formula for the simple interest rate for a 1-year loan.
Answer (B) is incorrect because it is the formula for the add-on installment interest for a 1-year loan. Answer (D) is
incorrect because the correct formula for the discounted interest rate is interest divided by the borrowed amount less
interest.

2(F)(T) Interest
C+ =
85 k Borrowed amount - interest .Answer (C) is correct. A time draft (trade acceptance) is a form of
commercial draft because it is drawn by a seller on the buyer; that is, it calls for the buyer to pay a specified amount.
The draft and the shipping documents related to the goods are then sent to the buyer's bank, which transmits the draft to
the buyer. The buyer accepts the draft by signing it. A time draft, however, is similar to a promissory note because it is
payable at a specific time in the future rather than upon acceptance by the buyer, which is characteristic of a sight draft.
If a seller is reluctant to ship goods because of concern about the buyer's ability to pay a time draft, the seller's bank
may, for a fee, guarantee payment. This banker's acceptance is an assumption of the obligation to pay at the due date.
Answer (A) is incorrect because a sight draft calls for immediate payment upon delivery of the shipping documents to,
and the acceptance of the draft by, the buyer. Answer (B) is incorrect because an open account is a credit arrangement
involving only the signing of an invoice by the buyer. Answer (D) is incorrect because the description is of a conditional
sales contract except that the seller, not the bank, retains title to the goods until the buyer has completed payment.

2(F)(T) Interest
C+ =
86 k Borrowed amount - interest .Answer (D) is correct. An unsecured loan is a loan made by a
bank based on credit information about the borrower and the ability of the borrower to repay the obligation. The loan is
not secured by collateral, but is made on the signature of the borrower. Unsecured credit is not backed by collateral.
Revolving credit, bankers' acceptances, lines of credit, and commercial paper are all unsecured means of borrowing.
Answer (A) is incorrect because a chattel mortgage is a loan secured by personal property (movable property such as
equipment or livestock). Also, a floating lien is secured by property, such as inventory, the composition of which may be
constantly changing. Answer (B) is incorrect because a chattel mortgage is a loan secured by personal property
(movable property such as equipment or livestock). Factoring is a form of financing in which receivables serve as
security. Answer (C) is incorrect because a chattel mortgage is a loan secured by personal property (movable property
such as equipment or livestock). Also, a floating lien is secured by property, such as inventory, the composition of which
may be constantly changing.

2(F)(T) Interest
C+ =
87 k Borrowed amount - interest .Answer (C) is correct. Commercial paper is the term for the
short-term (typically less than 9 months), unsecured, large denomination (often over $100,000) promissory notes issued
by large, creditworthy companies to other companies and institutional investors. In many instances, the maturity date is
only a few days after issuance.
Answer (A) is incorrect because an agency security is issued by a corporation or agency created by the U.S.
government. Examples are government securities issued by the bodies that finance mortgages, such as the Federal
National Mortgage Association (Fannie Mae). Answer (B) is incorrect because bankers' acceptances are drafts drawn on
deposits at a bank. The acceptance by the bank guarantees payment at maturity. They are normally used to finance a
specific transaction. Answer (D) is incorrect because a repurchase agreement involves a secured loan to a government
securities dealer. It allows the buyer to retain interest income although the seller-dealer can repurchase after a specified
time.

2(F)(T) Interest
C+ =
88 k Borrowed amount - interest .Answer (C) is correct. Commercial paper is a form of unsecured
note that is sold by only the most creditworthy companies. It is issued at a discount from its face value and has a
maturity period of 270 days or less. Commercial paper usually carries a low interest rate in comparison to other means
of financing. SMA 4M, Understanding Financial Instruments, observes that no general (active) secondary market exists
for commercial paper, but that "most dealers or organizations will repurchase an issue that they have sold."
Answer (A) is incorrect because commercial paper usually has a maturity date of 270 days or less to avoid securities
registration requirements. Answer (B) is incorrect because commercial paper is often issued directly by the borrowing
firm. Answer (D) is incorrect because interest rates must be higher than those of Treasury bills to entice investors.
Commercial paper is more risky than Treasury bills.

2(F)(T) Interest
C+ =
89 k Borrowed amount - interest .Answer (A) is correct. Commercial paper is a form of unsecured
note that is sold by only the most creditworthy companies. It is issued at a discount from its face value and has a
maturity period of less than 270 days. Commercial paper usually carries a low interest rate in comparison to other
means of financing.
Answer (B) is incorrect because only large companies with good credit ratings can find buyers for their commercial
paper. Answer (C) is incorrect because commercial paper is unsecured. Answer (D) is incorrect because investors must
pay a commission similar to that on other investment securities.

2(F)(T) Interest
C+ =
90 k Borrowed amount - interest .REQUIRED: The item not an advantage of using commercial
paper for short-term financing.
DISCUSSION: (C) Commercial payer is a short-term, unsecured note payable issued in large denominations by major
companies with excellent credit ratings. Maturities usually do not exceed 270 days. Commercial paper is a lower cost
source of funds than bank loans, and no compensating balances are required. Commercial paper provides a broad and
efficient distribution of debt, and costly financing arrangements are avoided. The market is not open to all companies
because only major corporations with high credit ratings can participate.
Answers (A), (B) and (D) are incorrect because lower rates, avoidance of compensating balance requirements, and
broad debt distribution are advantages of commercial paper.
2(F)(T) Interest
C+ =
91 k Borrowed amount - interest .Statement a is incorrect, and therefore the right answer.
Commercial paper is a type of unsecured promissory note issued by large, strong firms. Statements b, c, d, and e are all
accurate statements.

2(F)(T) Interest
C+ =
92 k Borrowed amount - interest .Answer (A) is correct. A short-term security issued by a
corporation or agency created by the U.S. government, such as the Federal Housing Administration, is an agency
security (agency issue). Among the largest issuers of agency securities (excluding the Treasury) are the Federal Home
Loan Banks, the Federal National Mortgage Association (Fannie Mae), and the other entities that provide credit to
farmers and home buyers. Other issuers of home mortgage-backed securities include the Government National
Mortgage Association (Ginnie Mae) and the Federal Home Loan Mortgage Corporation (Freddie Mac).
Answer (B) is incorrect because bankers' acceptances are issued by commercial banks to finance specific transactions.
Answer (C) is incorrect because commercial paper is a short-term, unsecured, promissory note issued by a commercial
enterprise. Answer (D) is incorrect because a repurchase agreement involves what is in essence a secured loan to a
dealer in government securities.

2(F)(T) Interest
C+ =
93 k Borrowed amount - interest .Answer (B) is correct. A document of title is usually issued by a
bailee covering goods in the bailee's possession or care (UCC 1-201). It represents ownership of the goods and is
ordinarily needed to obtain the goods from the bailee. The two major types of documents of title are bills of lading
(issued by carriers) and warehouse receipts. A warehouse receipt is issued by a person engaged in the business of
storing goods for hire. Security for short-term inventory financing can be arranged if the debtor places its inventory under
the control of the lender or its agent (e.g., a public warehouse), and the lender holds the warehouse receipts.
Answer (A) is incorrect because commercial paper is a type of unsecured, short-term promissory note issued by large
firms to other firms, insurance companies, mutual funds, etc. Answer (C) is incorrect because a revolving credit
agreement is a formal line of credit, usually with a bank, that large firms often use. Answer (D) is incorrect because a
line of credit is an arrangement, which may be formal or informal, between a commercial bank and its customer
concerning the maximum loan amount available.

2(F)(T) Interest
C+ =
94 k Borrowed amount - interest .Answer (B) is correct. The corporation can obtain trade credit for
20 additional days by not paying within the discount period. Instead of paying $99,000 to satisfy its obligation within 10
days, it can pay $100,000 at the end of 30 days. The corporation will thus incur $1,000 in interest to hold the $99,000 for
the 20 days. Because a 360-day year has 18 such periods, the interest rate is approximately 18.18% [($1,000/$99,000)
x 18]. However, if compounding effects are considered, the rate is higher. The effective rate, taking compounding into
consideration, is found using the following formula:
Effective rate = [1 + (1,000/99,000)]18- 1.0 = 19.83%.
In comparison, the 30-day note has an effective annual rate of 21.94%, calculated as follows:
[1 + (.20/12)]12 - 1.0 = 21.94%
Therefore, the corporation should use trade credit to obtain the short-term credit.
Answer (A) is incorrect because the effective trade credit rate is 19.83% depending on the method of calculation.
Answer (C) is incorrect because the note has an effective rate, including compounding effects, of 21.94%. The following
is the calculation: Effective rate = [1 + (.20/12)]12- 1.0 = 21.94%. Answer (D) is incorrect because the note has an
effective rate, including compounding effects, of 21.94%. The following is the calculation: Effective rate = [1 +
(.20/12)]12- 1.0 = 21.94%.
2(F)(T) Interest
C+ =
95 k Borrowed amount - interest .Answer (D) is correct. The most costly combination of
characteristics is a higher compensating balance and discount interest. The higher the compensating balance, the
higher the portion of the loan funds that must be left on deposit with the lender. Hence, the interest paid is charged on a
smaller amount of funds available to be used by the borrower, and the effective cost is higher. Also, discount interest is
deducted from the loan funds in advance, resulting in a further increase in the effective financing cost.
Answer (A) is incorrect because lower compensating balances and regular interest are less costly. Answer (B) is
incorrect because lower compensating balances and regular interest are less costly. Answer (C) is incorrect because
lower compensating balances and regular interest are less costly.

2(F)(T) Interest
C+ =
96 k Borrowed amount - interest .Answer (A) is correct. The most desirable set of terms are those
that result in the lowest cost of borrowing. Discount interest results in a higher effective borrowing cost than simple
interest because the bank deducts interest in advance so the borrower receives less than the face value of the loan. A
compensating balance results in a higher effective borrowing cost because the compensating balance is an amount of
cash that the firm is unable to use. The cheapest terms, given that all options have the same nominal interest rate, will
be simple interest with no compensating balance.
Answer (B) is incorrect because discount interest is disadvantageous to the borrower. Answer (C) is incorrect because
a compensating balance is disadvantageous to the borrower. Answer (D) is incorrect because discount interest and a
compensating balance are disadvantageous to the borrower.

2(F)(T) Interest
C+ =
97 k Borrowed amount - interest .Answer (A) is correct. For any given quoted nominal rate, the
least frequent compounding is associated with the lowest effective annual percentage cost. Annual compounding is less
frequent than semiannual, quarterly, or monthly. The term of the loan is not relevant to the calculation of the effective
annual percentage cost of financing.
Answer (B) is incorrect because, the more frequent the interest compounding, the more costly the loan. Semiannual,
quarterly, and monthly compounding are all more frequent than annual compounding. Answer (C) is incorrect because,
the more frequent the interest compounding, the more costly the loan. Semiannual, quarterly, and monthly compounding
are all more frequent than annual compounding. Answer (D) is incorrect because, the more frequent the interest
compounding, the more costly the loan. Semiannual, quarterly, and monthly compounding are all more frequent than
annual compounding.

2(F)(T) Interest
C+ =
98 k Borrowed amount - interest .Answer (D) is correct. If the company chooses the line of credit, it
will pay 17% interest on $80,000 ($100,000 - $20,000 discount) and 1% on the $20,000 unused portion, a total of
$13,800. The effective interest rate would thus be 17.25% ($13,800 ÷ $80,000).
Answer (A) is incorrect because if the company forgoes the cash discount, its effective rate is 25% ($20,000 ÷ $80,000
immediate cash price). Answer (B) is incorrect because an 18% effective rate exceeds that on the line of credit. Answer
(C) is incorrect because the effective rate would be 18.75% [(15% x $100,000) ÷ $80,000 available funds].

You might also like